Vous êtes sur la page 1sur 90

1.

Mobilization programs after flexor tendon repair not only promote gliding of the tendons and decrease adhesion formation but also: 1) have no effect on tendon healing. 2) increase loss of water and glycosaminoglycans. 3) increase collagen fibril size and improve tendon strength. 4) increase the likelihood of repair rupture. 5) lose repair strength over the first three weeks and must be carefully monitored. Preferred Response: 3 Discussion: It has been shown that gliding and stress of flexor tendons after repair not only keeps the tendon from losing strength postoperatively, as they do when kept immobile, but the healing milieu is markedly improved. Collagen fibril size increases and improves tendon strength. Water and glycosaminoglycan content increases and increases tendon strength. There is no decrease in tendon strength over the first three weeks atter repair, so there is no increase in tendon rupture. 2. A 28-year-old athletic woman with juvenile rheumatoid arthritis presents with a sudden loss of ring finger MCP extension, wrist pain, and the X-rays shown in Figure 1. Recommended treatment is: 1) Darrach procedure 2) extensor tendon grafting 3) tenodesis of the EDC-ring finger to the adjacent EDC 4) Suave-Kapandji procedure and EDC tenodesis 5) synovectomy DRUJ and extensor tendon reconstruction

Preferred Response: 4 Discussion: This patient with juvenile rheumatoid arthritis has developed a caput ulna syndrome with rupture of the extensor digitorum communis to the ring finger. Treatment should include tenodesis of the ruptured EDC to the adjacent extensor tendon. Extensor tendon grafting is less reliable and is reserved for cases in which three or more tendons are ruptured. Additionally, the patient's distal radioulnar joint subluxation should be treated. Synovectomy and retinacular reconstruction is usually unsuccessful because of the poor quality of the tissues which results in resubluxation. The Darrach procedure is the traditional treatment of choice, but can exacerbate ulnar subluxation of the carpus, which is already evident in this young patient. Also, rupture of the extensor tendons is a reported complication of the Darrach procedure. The Sauve-Kapandji operation is thought to offer additional support to the carpus and thus help to prevent this complication. 3. Following laceration and primary repair of the posterior interosseous nerve at the level of the proximal supinator the expected order of return of function is:

1) SUP, ECU, EDC, EDQ, EPL, EIP 2) SUP, EDU, EDQ, ECU, EPL, EIP 3) SUP, ECU, EDQ, EDC, EIP, EPL 4) SUP, EDC, ECU, EDQ, EPL, EIP 5) SUP, EDC, ECU, EDQ, EIP, EPL

Preferred Response: 4 Discussion: Understanding radial nerve motor branch anatomy is necessary for performing surgery in its vicinity, undertaking nerve repair, forecasting location of a compression lesion, and predicting rate and order of recovery. Radial nerve motor innervation order and its variability is controversial. Sunderland pointed out that accurate prediction of reinnervation order and rate depends not only on measurement of the motor branch points along the main nerve but also the additional distances along the branches to muscle entry points. Other studies that do not take this into account show different results. In the most recent study, Abrams et. al. utilizing 20 cadaver specimens found anatomic patterns that correspond to those of Sunderland, providing the most accurate and common order of innervation as illustrated in Figure 2. 4. Of the following muscles, which one is least affected by Volkmann's contracture?

1) Pronator teres. 2) Flexor digitorum profundus. 3) Flexor pollicis longus. 4) Flexor carpi ulnaris. 5) Flexor digitorum superficialis Preferred Response: 4 Discussion: In Volkmann's contracture the most severely affected muscles are the flexor digitorum profundus and flexor pollicis longus, followed by the flexor digitorum superficialis and pronator teres. There is much less injury to the wrist flexors and extensors and brachioradialis. 5. A 45-year-old female sustained a puncture wound to her palm from a cactus plant. Five days later, she developed an ulceration at the puncture site and Iymphangitis with nodules along the volar forearrn. Two weeks later, these nodules ulcerate. Cultures should be taken and antibiotic treatment should include: 1) Dicloxacillin 500 mg po q six hours 2) Cefazolin I gm IV q eight hours 3) Acyclovir 400 mg po q eight hours 4) Potassium Iodide .3 mg po tid 5) Amphotericin B 50 mg IV q day Preferred Response: 4 Discussion: Fungi are often overlooked as a source of hand infections. Sporotrichosis is the most common subcutaneous fungal infection found in North America and predominately affects the upper extremities. Implantation can be caused by a variety of plants in addition to the frequently recognized rose thorn, including cactus and sphagnum moss. The infection is difficult to diagnose on the basis of standard stains and definitive diagnosis

requires cultures. Treatment of choice is a saturated solution of potassium iodide (Igm/ml) taken .3 mg po tid after meals and increased as tolerated to 1.5 gm po tid. 6. The Jamar dynamometer five-rung test: 1) Permits subjective assessment of the motivation and sincerity of effort of the patient. 2) Is not a reliable test because it is dependent on patient motivation. 3) Cannot be used to accurately assess patients suspected of low sustained grip effort. 4) Shows flatter curves for sincere effort. 5) Rarely results in bell-shaped curves in strong patients. Preferred Response: 1 Discussion: A number of methods to detect patients who purposely exert low effort during group evaluations have been devised. The Jamar dynamometer has been shown using a five-rung test to demonstrate objective answers to the subjective questions of motivation and effort. A bellshaped curve is correlated with appropriate effort and a flatter curve with feigning a response.

7. A 46-year-old male factory worker who performs martial arts as an avocation presents with a four-month history of wrist pain. Examination reveals dorsal wrist synovitis and tenderness. Radiographs are shown in Figures 3A and 3B. The most likely diagnosis is: 1) Kienbock's disease 2) Preiser's disease 3) Accessory ossicle of lunate 4) Volar fracture dislocation of lunate 5) Lunate fracture

Preferred Response: 5 Discussion: Although rare, fresh lunate fractures, as distinguished from Kienbock's disease, do occur. Teisen and Hjarbaek divided these injuries into five groups based on the location of the fracture. Because important nutrient vessels and stabilizing ligaments attach to the palmar pole of the lunate, displaced fractures of the palmar pole and transverse fractures through the mid portion would appear to be best treated by open reduction and internal fixation. Minimally displaced fractures of the dorsal pole, sagittal fractures, and small marginal fractures should be treated non-surgically.

8. Two years after traumatic brain injury, a 37-year-old male with a severe spastic flexion contracture of the hand and wrist undergoes STP transfer, wrist flexor tenotomy, and wrist arthrodesis. Postoperatively he has persistent MP flexion contractures. What is the most likely cause of this deformity? 1) Inadequate lengthening with the STP transfer. 2) Failure to release contracted MP joint capsules. 3) Unopposed lumbrical pull. 4) Failure to release the intrinsics. 5) Failure to excise the proximal carpal row when performing the arthrodesis.

Preferred Response: 4 Discussion: Severe spastic flexion contractures of the hand and wrist may benefit from STP transfer, FPL lengthening and wrist arthrodesis with or without proximal row carpectomy. Intrinsic tightness may be responsible for MP contractures. Intrinsic release or neurectomy of the motor branch of the ulnar nerve will help prevent or correct this deformity. 9. Intrinsic spasticity of the fingers and thumb after closed head injury results in an inability to: 1) Adduct the thumb. 2) Extend the interphalangeal joint of the thumb. 3) Flex the metacarpophalangeal joint of the fingers. 4) Extend the metacarpophalangeal joints of the fingers. 5) Extend the proximal interphalangeal joints of the fingers. Preferred Response: 4 Discussion: Intrinsic tightness of the thumb results in a position of metacarpal adduction (from tightness of the adductor pollicis, first dorsal interosseous and thenar musculature), MCP joint flexion (from tightness of the thenar muscles) and narrowing of the first web space. Extrinsic FPL tightness produces IP flexion deformity of the thumb. Intrinsic tightness of the fingers results in a position of fixed flexion of the MCP, and may lead to the development of swan-neck deformity at the PIP and DIP joints.

10. A 45-year-old farmer "jams" his non-dominant ring finger. He complains of pain and modest swelling about the PIP joint that gradually resolves. Examination six weeks later reveals a 30 extensor lag at the PIP joint. Active flexion is normal and passive motion is 0 to 95. Recommended treatment is:

1) PIP joint extension splint for six weeks. 2) Central slip repair. 3) Buddy taping of ring finger to long finger. 4) Arthrodesis of PIP joint. 5) Lateral band release. Preferred Response: 1 Discussion: The patient sustained a closed injury to the central slip when he jammed his finger. Splinting the PIP joint in full extension, allowing active and passive DIP motion, is likely to allow healing of the central tendon rupture with appropriate gliding of the lateral bands. Only if this first line conservative treatment fails should operative intervention be considered. Primary shortening of the central tendon can be considered as could division of the extensor mechanism over the middle phalanx. Lateral band release in the face of full passive range of motion is not needed to obtain motion and will remove a portion of the extensor mechanism needed for PIP and DIP motion. Buddy taping is recommended for PIP collateral ligament injuries but is not helpful for extensor injuries 11. A 56-year-old patient with a 42-week history of upper extremity pain is suspected of having reflex sympathetic dystrophy. The most accurate way to make a diagnosis of RSD is: 1) clinical history and physical examination 2) three-phase bone scan 3) intravenous phentolamine test 4) thermography 5) isolated cold-stress testing Preferred Response: 1 Discussion:

The most accurate way to make a diagnosis of RSD is on the basis of the clinical history and physical exam. A 1984 report by Mackinnon and Holder implied that a negative three-phase bone scan essentially ruled out RSD. Subsequent reports have challenged this concept. Many patients with clinical RSD particularly for more than 26 weeks may have a negative three-phase bone scan. Thermography, isolated cold-stress testing, and intravenous phentolamine are all considered non-specific tests. 12. The artery of origin of a dorsoradial pedicled bone graft for the management of scaphoid nonunions is: 1) posterior interosseous 2) dorsal intercarpal arch 3) posterior division of anterior interosseous 4) radial 5) ulnar Preferred Response: 4 Discussion: Both Zaidenberg et. al. and Sheetz et. al. describe a branch of the radial artery that provides intraosseous blood supply to the dorsoradial aspect of the distal radius. Sheetz et. al. describe it as the 1, 2 intercompartmental supraretinacular artery (I, 2 IC SRA) and Zaidenberg et. al. as the first ascending irrigating branch of the radial artery that arises at the radiocarpal joint, passes deep to the first dorsal compartment, and then goes retrograde to lie on the periosteum between the first and second dorsal compartments where it sends nutrient vessels to the radius. Sheetz et. al. found that the 1, 2 IC SRA arose 48 mm proximal to the radiocarpal joint, went superficial to the retinaculum between the first and second dorsal compartments and then went distally deep to the extensor tendons to joint either the radial artery or the dorsal radiocarpal arch. Both authors describe a pedicled bone graft that can be used in the management of scaphoid nonunions based on

this branch of the radial artery. 13. A seven-year-old spastic diplegic has the deformity pictured in Figure 4. He cannot actively extend the wrist but can easily be passively corrected to 20 of dorsiflexion. He has poor grasp release using a tenodesis effect with the wrist flexed and cannot release with the wrist extended beyond neutral. The patient can voluntarily flex the fingers but the first ray is fixed in an adduction contracture and the IP joint of the thumb does not actively extend. The recommended surgical plan would involve: 1) wrist arthrodesis in neutral and palmaris longus transfer to the EPL 2) FCU to ECRB tendon transfer 3) adductor policis brevis release with reverse radial forearm flap for the first web space and FCR to EDC transfer 4) adductor pollicis brevis release, PL to EPL transfer, FCU to ECRB, PT to EDC 5) proximal row carpectomy, adductor release and PL to EPL

transfer Preferred Response: 4 Discussion: The clinical presentation of this child demonstrates three

problems: wrist flexion, inadequate release and a "thumb-in-palm" deformity characterized by a fixed adduction contracture of the first metacarpal and a weak EPL. Wrist arthrodesis is usually reserved for severe deformities in adult patients where hygiene is of concern and there is little function. This patient's wrist can be passively corrected beyond neutral and he relies upon a tenodesis effect to assist release. A fused wrist would compromise this function. Proximal row carpectomy, as described by Omer, is also reserved for severe cases of wrist flexion and is not required in this patient who is passively correctable to neutral. The Green transfer of the FCU to the ECRB is the classic transfer described for wrist flexion and weak grasp. While appropriate for this patient's wrist flexion deformity, it alone will worsen the release problem and not address the thumb-in-palm deformity. This transfer is assisted by the improved finger extension from the pronator teres to EDC transfer. Treatment of the first ray abduction contracture involves release of the adductor and possibly the flexor pollicis brevis muscles. A reverse radial forearm flap is not required. Transferring the palmaris longus tendon to the weak EPL will augment interphalangeal extension . 14. A 39-year-old white male jams his long finger while playing basketball. Radiographs of his injured finger are shown in Figures 5A and 5B. Recommended treatment is: 1) Extension block splinting with intermittent active motion. 2) Open reduction internal fixation. 3) Splint for three weeks, then buddy tape. 4) Volar plate arthroplasty. 5) Dynamic skeletal traction.

Preferred Response: 5 Discussion: Dynamic skeletal traction is favored in this case because of involvement of the volar 50% of the middle phalanx articular surface as well as the dorsolateral quadrant (implied by angulation in the PA X-ray). This puts the fracture more in the category of a pilon fracture that Stern and coworkers say is more safely treated with dynamic traction. Splinting alone will not prevent progressive collapse due to longitudinal force. Because of the asymmetrical nature of the articular surface fracture, volar plate arthroplasty would not be suitable. Open reduction and internal fixation is technically difficult, carries additional morbidity, and would probably require external fixation to neutralize longitudinal forces. 15. In a skeletally immature individual, growth abnormalities caused by frostbite are a consequence of injury to: 1) vascular endothelium 2) secondary spongiosa 3) chondrocytes 4) osteocytes 5) perichondrial ring of LaCroix Preferred Response: 3 Discussion: The major lesion responsible for growth abnormalities in the juvenile hand is direct injury to chondrocytes in the growth plate resulting in premature physeal closure. Direct injury to vascular endothelium also occurs; however, it is not a major mechanism for skeletal abnormalities in the skeletally immature. Vascular injury is more responsible for soft-tissue necrosis. Injury to osseous cells does occur; however, new cells are recruited and function in a normal fashion.

16. Which of the following procedures is most appropriate for permanent ablation of intrinsic function in a patient with spastic paralysis of the hand and upper extremity after closed head injury? 1) Ulnar nerve neurectomy in the forearm. 2) Ulnar nerve neurectomy in Guyon's canal. 3) Neurectomy of the motor branch of the ulnar nerve in Guyon's canal. 4) Neurectomy of the motor branch of the ulnar nerve to the adductor pollicis. 5) Phenol block of the motor branch of the ulnar nerve in Guyon's canal. Preferred Response: 3 Discussion: Spastic paralysis of the intrinsic muscles of the hand limits metacarpophalangeal joint extension, thumb extension, and anteposition. Complete ulnar nerve neurectomy in the forearm or in Guyon's canal will dennervate the intrinsic muscles, but will also result in an unacceptable loss of sensation. Neurectomy of the motor branch of the ulnar nerve to the adductor pollicis will not dennervate the interosseous or hypothenar muscles. Phenol block of the ulnar nerve in Guyon's canal may be used as a temporizing procedure when multiple procedures are planned and the final muscle balance is uncertain. It may also be used to prevent contractures and aid splinting when further neurologic recovery may still occur. 17. A 59-year-old patient has a mucous cyst on the dorsum of his index finger at the ulnar side of the distal interphalangeal joint near the nail matrix. The nail has a longitudinal groove adjacent to the cyst. During surgical excision which of the following maneuvers will most effectively reduce the risk of recurrence? 1) Removal of the nail

2) Ablation of the nail bed 3) Capsular excision 4) Osteophyte excision 5) En bloc excision of the cyst and overlying skin Preferred Response: 4 Discussion: Without excision of the osteophyte underlying the osteophyte it will recur in most, if not all, of the cases. The cyst is not always seen on X-ray, but is present and underlies almost every true mucous cyst. Oblique X-rays are often helpful in confirming its presence prior to surgery.

18. The most common hand deformity in juvenile rheumatoid arthritis is: 1) Loss of wrist flexion 2) Radial deviation of the digits at MPJ 3) Synovitis and laxity of thumb joint 4) Ulnar deviation of the carpus 5) Loss of flexion at MPJ and PIPJ Preferred Response: 1 Discussion: The classic early wrist deformity in juvenile rheumatoid arthritis is loss of wrist extension with a progressive flexion deformity, emphasizing the need for wrist extension splinting as these joints usually fuse spontaneously. Young patients with JRA (less than four years of age) show radial deviation of the wrist, ulnar deviation of the MPJ, and flexion deformities of the fingers. By age six it shifts to the classic JRA presentation of ulnar deviation of the carpus and metacarpal radial deviation of the digits at MPJ and loss of digit flexion at MPJ and PIPJ. The thumb shows frequent involvement with synovitis, ligament laxity, and joint

erosions of the CMC, MP, and IP joints.

19. A 26-year-old woman, under custodial care, presents to the hand clinic with a five-week history of pain and swelling in her index finger. Nursing home records indicate a history of trauma but no additional history is available. X-rays ( Figures 6A and 6B) are obtained. You recommend: 1) curettage, isoniazid, and rifampin 2) index-ray amputation 3) curettage and cephalosporin 4) biopsy and radiation/chemotherapy

5) curettage and bone graft Preferred Response: 3 Discussion: This patient has osteomyelitis. The X-rays suggest that she had a previous fracture as evidenced by pin tracks. The radiographic hallmarks of osteomyelitis include bone formation and resorption. In this case, resorption is evidenced by the two radiolucencies. Bone formation is evidenced by periosteal reaction and a sequestrum (density) seen in the distal lucency. In addition there often is soft-tissue swelling. Osteomyelitis in the tubular bones of the hand occurs most commonly after open fractures ( I % to 11 %) and in fractures that are operatively fixeM. Direct extension from a soft-tissue infection can also

cause osteomyelitis. S. aureus is the most common organism. Granulomatous infections of bone are rare and usually occur in immunocompromised patients. A primary or metastatic malignancy is rare in this age group. 20. When calcium gluconate is used to treat a hydrofluoric acid burn, its mechanism of action is: 1) local vasodilatation resulting in dilution of tissue acid levels 2) formation of insoluble complexes of calcium and fluoride ions 3) bind hydrogen ions halting tissue necrosis 4) to prevent free radical formation 5) increased calcium levels that buffer the acidic environment Preferred Response: 2 Discussion: Hydrofluoric acid is an inorganic acid commonly used as an industrial solvent in glass and metal cleaning and is used extensively in the petroleum industry. Cutaneous exposure cause severe burns characterized by intense pain. The fluoride ion readily penetrates the skin and binds with tissue calcium. This in turn causes a release of intra-cellular potassium that results in disruption of normal cellular processes. The increased potassium causes intense nerve stimulation leading to extraordinary pain. Calcium gluconate binds free fluoride ion and forms an insoluble complex that prevents the binding of tissue calcium. This in turn prevents the depletion of tissue calcium and the secondary release of intra-cellular potassium. 21. A 65-year-old female sustains a fracture/dislocation of her right elbow. Three months later she complains of persistent elbow stiffness and is unable to reach her mouth with her hand. Examination reveals 50 to 80 of active motion with 80 of pronation and 70 of supination 30. A lateral X-ray is seen in Figure 7. Recommended treatment is: 1) Immediate excision of HO, capsulectomy, low-dose radiation.

2) Bone scan and alkaline phosphate. 3) Indocin, low-dose radiation, physical therapy. 4) Excision of HO, capsulectomy, low-dose radiation in three months. 5) Physical therapy.

Preferred Response: 4 Discussion: This patient is disabled enough with elbow stiffness that limits self-care activities about her face and head. Surgical intervention to increase range of motion is definitely indicated. Timing is the critical issue with regard to heterotopic ossification. It is generally accepted that ablation of disabling HO about the upper extremity in the burned or brain-injured patient can safely be carried out after one year. This somewhat arbitrary period of delay factors in "maturity" of the lesion as well as potential neurologic recovery in the head-injured patient or local stability in the burn victim. Jupiter advocates earlier intervention on the patient without neural axis trauma. In the absence neural axis trauma and/or burns, Hastings and Graham advise deferral of surgical intervention until six months have elapsed from initial trauma. Bone scan and alkaline phosphate have been advocated as indicators of the maturity of the lesion but are not always dependable.

22. Myofibroblasts are the predominant cell type in: 1) Dupuytren's nodules and healing epidermal wounds.

2) Dupuytren's bands and full-thickness healing open wounds. 3) Dupuytren's nodules and full-thickness healing open wounds. 4) Dupuytren's cords and healing epidermal wounds. 5) Epibolic cell migration and healing epidermal wounds Preferred Response: 3 Discussion: The myofibroblast has been studied in Dupuytren's contracture. In order to detect the myofibroblast, electron microscopes are used. Such studies have demonstrated the myofibroblast to be present in the nodular tissue of Dupuytren's contracture but not in the cord-like structures. Myofibroblasts have been found in a full-thickness wound between the 10th and 20th days after wounding. This time period also coincides with maximum speed of wound contraction. Epibolic cell migration is found in epidermal wound repair. Epithelial cells migrate from the edge of the wound, populating the wound surface, forming a monolayer of cells at the leading edge.

23. The nerves most at risk for injury during surgical release of the first dorsal compartment are: 1) Dorsal sensory radial and posterior interosseous 2) Lateral antebrachial cutaneous and palmar cutaneous 3) Palmar cutaneous and dorsal sensory radial 4) Lateral antebrachial cutaneous and posterior interosseous 5) Lateral antebrachial cutaneous and dorsal sensory radial Preferred Response: 5 Discussion: The lateral antebrachial cutaneous and dorsal sensory radial nerves merge and intermingle over the first dorsal compartment. The posterior interosseous nerve is located between the third and fourth dorsal compartments. The palmar cutaneous nerve is

located between the flexor carpi radialis and the palmaris longus.

24. A patient has a release of the Al pulley of the thumb. The adjacent structure most in jeopardy of operative injury is: 1) Abductor pollicis brevis 2) Ulnar digital nerve 3) Adductor pollicis 4) Flexor pollicis longus 5) Radial digital nerve Preferred Response: 5 Discussion: The radial digital nerve crosses the flexor tendon sheath just proximal to the Al pulley and just underneath the skin. It is the anatomic structure most in danger of iatrogenic injury during this procedure.

25. A 19-year-old college student sustained a closed transverse fracture of the mid-shaft of the proximal phalanx of his middle finger. Radiographs reveal apex volar angulation of 40. A closed reduction is performed and a functional cast brace applied. One week later clinical examination and radiographs reveal complete recurrence of the initial deformity. In the operating room, you are again able to achieve an excellent closed reduction of the fracture. You would now recommend fixation with: 1) Two percutaneous transverse Kirschner wires 2) Two percutaneous intramedullary Kirschner wires 3) Percutaneous screws 4) Dorsal mini- or micro-plate 5) Lateral mini- or micro-plate

Preferred Response: 2 Discussion: Percutaneous intramedullary Kirschner wires or their equivalents have set the standard among the available choices for treatment of closed reducible transverse diaphyseal proximal phalangeal fractures. Transverse Kirschner wires are not appropriate for a transverse fracture pattern. Since the fracture can be reduced by closed manipulation, the additional risks of an open procedure are not justified. 26. A 25-year-old right-handed accountant suffers dorsal fracture dislocation of the PIP joint of the right middle finger. Closed reduction is easily achieved but the joint subluxes with extension beyond 25. It is stable and congruent from 25 to 95 of flexion. Appropriate management includes: 1) static splinting in 30 of flexion for three weeks. 2) early ROM exercises in a 30 extension block splint. 3) an Eaton volar plate arthroplasty. 4) open reductions and internal fixation. 5) percutaneous pinning in 25 of flexion for three weeks. Preferred Response: 2 Discussion: Elevation is always essential following significant PIP joint injury to avoid swelling and minimize stiffness. Splinting the joint in 30 of flexion for three weeks will almost certainly result in a flexion contracture and it can take many months to regain motion. Eaton volar plate arthroplasty is indicated when accurate congruous reduction can not be obtained. Use of the extension block splint (figure-of-eight type works well) will allow the patient to regain flexion while healing occurs. A probable flexion contracture of 30 or less is acceptable from a functional point of

view but greater than this is not. Therefore you don't want to use this technique if 40 to 45 of flexion is needed to keep the joint reduced. The advantage of pinning the joint is that the reduction can be maintained however, exercises cannot be started. 27. Late sequelae of frostbite may include which of the following findings? 1) Anesthetic digits 2) Dry skin 3) Decreased nail and hair growth 4) Increased digital warmth 5) Raynaud's phenomenon Preferred Response: 5 Discussion: Late sequelae of frostbite include vasospastic syndromes secondary to increases sympathetic tone. Symptoms and signs include heightened pain upon cold exposure, hyperhidrosis, trophic changes (increased nail and hair growth) and Raynaud's phenomenon.

28. Which of the following is true with regard to the serratus anterior muscle free flap? 1) It is innervated by the serratus branch of the thoracodorsal nerve. 2) It may be raised as a muscluocutaneous flap. 3) It can be used as a functional muscle transfer. 4) Its use is limited by a short (6 cm) vascular pedicle. 5) The donor site usually requires skin grafting. Preferred Response: 3 Discussion:

The serratus anterior muscle may be transferred as a free-flap for small upper extremity defects. The flap has a long vascular pedicle that can be lengthened to 15 centimeters by dividing the thoracodorsal artery and vein and dissecting the subscapulo arterial system in the axilla. The serratus anterior muscle is innervated by the long thoracic nerve. It can be used as a functional muscle transfer for the upper extremity or hand. The cutaneous territory of the serratus flap has not been defined. The donor site is always closed primarily.

29. A 30-year-old male presents with the abnormality seen in Figure 8. It has gradually enlarged over 10 years, has been intermittently painful and has ulcerated on several occasions. The physician should recommend: 1) Angiographic evaluation with gelfoam and irradiation. 2) Ray resection. 3) Resection of all abnormal vessels, and vein grafting of the digital artery. 4) Combined embolization and surgical resection. 5) Observation.

Preferred Response: 5 Discussion:

This is a low flow venous malformation. Although most continue to enlarge and cause a variety of symptoms including pain, ulceration, and limited motion surgical treatment is recommended only in certain instances. Recurrence is probable following resection, and embolization and irradiation are of no benefit. This patient's symptoms are relatively mild and treatment is not necessary. 30. A 35-year-old skier falls and injures her wrist. She cannot remember the exact mechanism of injury. The scaphoid and lunate are flexed and the triquetrum is extended on a standard lateral radiograph. These findings are most consistent with: 1) Scapholunate dissociation 2) Perilunate dislocation 3) Lunate dislocation 4) Lunotriquetrial dissociation 5) Mid-carpal dislocation Preferred Response: 4 Discussion: Scaphoid and lunate flexion and the triquetral extension clearly define the carpal malalignment that occurs with a complete triquetral-lunate ligament tear and consequent dissociation. 31. Prolonged pressure on a peripheral nerve leads to demyelination, usually followed by remyelination. There is resultant slowing of nerve conduction over the involved segment. This is due to: 1) Impaired axoplasmic transport. 2) Shorter internodal distances with remyelination. 3) Thinner layer of myelin. 4) Endoneurial edema. 5) Intussusception of successive nodes.

Preferred Response: 2 Discussion: Demyelination occurs when prolonged pressure on a nerve leads to intussusception of successive nodes of Ranvier. When the myelin sheath reforms, the internodal distances are decreased (i.e. the nodes are closer together). Since saltatory conduction occurs between nodes a shorter distance between successive nodes will lead to a reduction in conduction velocity. 32. A 60-year-old male complains of wrist pain and weakness of grip eight months after a resection of the distal ulna with ECRL to ECU tendon transfer. Pre- and postoperative radiographs are shown in Figure 9A and Figure 9B. This sequence of events is primarily due to: 1) Resection of the distal ulna 2) Attenuation of the radiocarpal ligaments 3) Attenuation of ulnocarpal ligaments 4) ECRL to ECU tendon transfer 5) Rupture of TFC

Preferred Response: 2 Discussion: Contrary to previous teaching, ulnar translation of the carpus is primarily due to radiocarpal ligament attenuation. In this case, the

inflammatory arthritis caused progressive ligamentous laxity and attenuation with loss of articular cartilage that then led to ulnar translocation of the carpus. Had this potential for collapse been recognized by the surgeon at the time of distal ulnar resection, a simultaneous radiolunate fusion would have prevented the ulnar translocation. 33. Which of the following is the most appropriate treatment for a four-year-child who presents with a Wassel Type IV thumb duplication with similar appearing duplicate thumbs? 1) Retention of the radial-most digit with ulnar collateral ligament reconstruction. 2) Retention of the ulnar-most digit with radial collateral ligament reconstruction. 3) The Bilhaut-Cloquet sharing procedure. 4) Observation until skeletal maturity followed by retention of the ulnar-most digit with radial collateral ligament reconstruction. 5) Resection and "on-top" pollicization. Preferred Response: 2 Discussion: A Wassel Type IV thumb duplication consists of duplication distal to the metacarpal level. If the digits are similar in bony and soft-tissue structure, the ulnar most digit is most commonly retained secondary to the importance of a stable ulnar collateral ligament. Retention of the radial most digit is more hypoplastic. The Bilhaut-Cloquet sharing procedure is more appropriate in Wassel Type I (bifid distal phalanx) or Type Wassel Type II (duplicate distal phalanx) deformities. There is no reason to wait until skeletal maturity to reconstruct a duplicate thumb.

34. Which of the following is true regarding phosphorus burns of the hand? 1) Phosphorus ignites spontaneously when exposed to water.

2) Phosphorus may be found in insecticides and fertillzers. 3) Emergency treatment includes the application of calcium gluconate gel. 4) Surgical debridement is not recommended. 5) Acute washing with copper sulfate represents definitive treatment. Preferred Response: 2 Discussion: Although phosphorus burns are more common in military personnel, they do occur in civilians usually from the phosphorus in fireworks, insecticides, rodent poisons, and fertilizers. Phosphorus ignites spontaneously when exposed to air and is rapidly oxidized to phosphorus pentoxide. It is extinguished by water, but may reignite upon drying. Emergency treatment consists of copious water irrigation and debridement of visible particles. The wound should be briefly washed with 1% copper sulfate to form black cupric phosphide and facilitate the removal of phosphorus particles. Copper sulfate is not a form of treatment and should be washed off to prevent copper toxicity after all particles have been removed. Calcium gluconate is used in the treatment of burns caused by hydrofluoric acid, not phosphorus.

35. What is the effect of dextran on the coagulation system. 1) Interferes with platelet aggregation. 2) Increases the activity of antithrombin III. 3) Catalyzes the conversion of plasminogen to plasmin. 4) Serves as a plasma expander. 5) Interferes with Vitamin K. Preferred Response: 4 Discussion: Dextran is a partially hydrolyzed polymer obtained from the

bacterium Leuconostoc mesenteroides. It serves as a plasma expander and increases colloid osmotic pressure. It may be dangerous to patients with CHF or decreased renal function. Dextran has no in vitro effect on platelet function. Heparin binds to antithrombin III and increases its activity 1000 fold. Coumadin interferes with Vitamin K metabolism. 36. A 39-year-old cook presents with a volar dislocating distal radioulnar joint after a fall. A CT scan demonstrates no bony injury to the distal radioulnar joint. The injury most likely responsible for the DRUJ instability is a tear of the: 1) Extensor carpi ulnaris tendon sheath. 2) Triangular fibrocartilage disk. 3) Triangular fibrocartilage from its ulnar attachment. 4) Triangular fibrocartilage from its radial attachment. 5) Ulnocarpal ligaments. Preferred Response: 3 Discussion: Although several structures can be injured, a tear of the triangular fibrocartilage from its ulnar attachment is the principle lesion associated with acute traumatic distal radioulnar joint instability. With increasing severity of injury, there is concomitant injury to the adjacent extensor carpi ulnaris tendon subsheath, the ulnocarpal ligaments, the lunotriquetral interosseous ligament, and the triquetral-capitate and triquetral-hamate ligaments.

37. Which treatment is appropriate for the reconditioning phase of a patient suffering from lateral epicondylitis? 1) Maintenance program of stretching, eccentric strengthening, and ice massage. 2) Modalities of ultrasound, phonophoresis, and iontophoresis (I ml Lidocaine, 2 ml dexamethazone). 3) Daily static splinting and anti-inflammatory medicatlon.

4) Open revision of dorsal extensor forearm musculature, common origin, and associated aponeuroses. 5) Application of tennis elbow strap. Preferred Response: 1 Discussion: Three specific treatment phases have been described for lateral epicondylitis. Phase I involves reduction of inflammation, pain modulation, and release of any underlying adhesions in fascia and trigger points, and increasing flexibility. Phase 2 consists of initiating eccentric strengthening exercises using one- to twopound weights, increasing to three- to five-pound weights, 10 to 30 repetitions. The objective of this phase is to prevent recurrence of progressive overload of extensor tendons. The third phase focuses on reconditioning and functional return to prior levels of activity. Functional strengthening programs should be closely monitored and upgraded accordingly utilizing a work simulator. In this phase, the importance of stretching, eccentric strengthening, and ice massage cannot be overemphasized. Electrical modalities such as ultrasound, phonophoresis, and iontophoresis should be trialed during Phase I for six to eight applications before evaluating a particular modality's contribution to healing the injured tissue. Indications for surgery as a treatment for lateral epicondylitis coincide with between 3% and 8% of all reported cases where symptomatology persists for a duration of at lease six months with failed conservative treatment and absence of cervical or intra-articular signs or symptoms. A counter force tennis elbow strap may be used during Phase 3 as an adjunctive treatment to stretching, eccentric stretching, ice massage, and specific instructions in maintenance program. 38. A 27-year-old female complains of difficulty gripping eight months following primary repair of a tidy laceration of the flexor tendons in Zone 2 of the ring finger. Specific active ranges are as follows (with passive ranges noted in parentheses): MP - 0/90; PIP - 0/60 (100); DIP - 0/40 (70). Her finger motion has not changed for three months despite intensive physical therapy.

Recommended treatment ls: 1) Dorsal IPJ capsulotomies. 2) Flexor tenolysis. 3) Staged flexor tendon reconstruction. 4) Continued occupational therapy. 5) Extensor tenolysis. Preferred Response: 2 Discussion: The limited active flexion with full passive flexion indicates flexor tendon adhesions following tendon repair. Since it is more than three to six months postrepair and a plateau has been reached with regard to recovery of active flexion, flexor tenolysis is indicated to restore greater active motion. Continued therapy is unlikely to help. Since she has good passive flexion, dorsal capsulotomy is unnecessary and there is no indication for extensor tenolysis. Staged flexor tendon reconstruction might be a consideration for the salvage of a severely damaged flexor tendon system, but not the primary treatment for this supple finger with intact, but limited, flexor tendon pull through.

39. A 60-year-old right-handed female has Stage IV osteoarthritis of her right thumb CMC joint, an adduction contracture of the first metacarpal, and a 40 hyperextension deformity of her thumb MP joint. In addition to a thumb CMC joint arthroplasty, treatment should include: 1) Adductor pollicis muscle release 2) First web release by Z-plasty 3) Flexor pollicis longus Z-lengthening 4) MP joint stabilization 5) IP joint fusion Preferred Response: 4

Discussion: Hyperextension of the MP joint can occur as thumb CMC arthritis progresses. This occurs as the result of progressive MP joint capsular laxity and the unopposed extension force by the extensor pollicis brevis. It is aggravated as the first metacarpal becomes fixed in the flexed and adducted position with progression of the thumb CMC arthritis. The MP hyperextension deformity increases the stress on the CMC joint, and may cause the CMC arthroplasty to fail if it is not corrected. Fusion of the MP joint in 15 to 25 of flexion will permanently stabilize the MP joint, protect the CMC joint arthroplasty, and interfere minimally with thumb function. Soft-tissue stabilization may be adequate for less severe MP joint hyperextension .

40. Initial postoperative therapy following repair of a Zone II flexor tendon in a six-year-old boy should include: 1) Kleinert early passive mobilization. 2) Early active place/hold mobilization. 3) Early active mobilization. 4) Immobilization. 5) Duran/Houser early passive mobilization. Preferred Response: 4 Discussion: Immobilization would be the best choice in treating this case. Such a technique is still the treatment of choice for patients under 10 years of age, those with cognitive deficits, or those where compliance/participation is questionable. Immobilization is necessary to protect the repair in such populations or to protect other structures surrounding the repair. The immobilization phase continues from the time of repair until three to four weeks postoperatively, after which mobilization techniques are initiated. Both the Kleinert and the Duran/Houser early passive mobilization protocols are appropriate treatment options in a

reliable adult patient. The patient must demonstrate excellent understanding and awareness of the rationale and must employ excellent independent technique to prevent tendon ruptures. Early active mobilization and early active place/hold mobilization techniques are also appropriate for recently injured tendons. However, similarly, such treatment would not be applicable in a six-year-old boy.

41. A 66-year-old female suffered an intra-articular fracture of her right distal radius with much dorsal comminution. The junior orthopaedic resident performed an initial reduction and re-established anatomic alignment. She is placed in a long-arm cast with good molding that extends from the digital crease to above the elbow. She was sent home with a sling and told to exercise her fingers. The most likely long-term complication of this treatment if not modified promptly is: 1) limited MCP flexion. 2) painful flexion and extension of the wrist. 3) cast sores 4) limited pronation and supination of the forearm. 5) carpal tunnel syndrome. Preferred Response: 1 Discussion: As long as the reduction remains anatomic, no additional immediate attention needs to be paid to the fracture. However, in this type of severe fracture, with elevation limited by the sling, swelling is likely to occur. The MCP joints are being held in full extension by the cast which extends to the mid proximal phalanx (MP flexion crease), it should extend only to the distal palmar crease. This combination of swelling and MCP joints immobilized in extension will commonly lead to MCP joint extension contractures with severe functional consequences. Although carpal tunnel syndrome may occur following distal radius fractures, nothing in the treatment described above would

be more likely to cause it. 42. A 20-year-old graduate student caught her thumb in a centrifuge and complains that it went "out of place." Examination reveals a hyperextension deformity of the MCP joint, flexion at the IP joint, intact skin, and no neurovascular deficits. Radiographs of her thumb are shown in Figure 10. Treatment should consist of: 1) Closed reduction by distraction, hyperextension, and then flexion of the MCP joint. 2) Closed reduction by pressure over the dorsum of the base of the proximal phalanx, and then flexion. 3) Volar incision, intrinsic release, reduce the proximal phalanx. 4) Volar incision, retrieve the volar plate, and reduce the proximal phalanx. 5) Dorsal incision between the extensor pollicus longus and brevis, vertical incision in the middle of the volar plate, reduce the proximal phalanx.

Preferred Response: 2 Discussion: This is a simple dislocation. The volar plate is not interposed between the dorsum of the metacarpal head and the base of the

proximal phalanx. Pressure over the base of the proximal phalanx and flexion will achieve reduction. Distraction and hyperextension will flip the volar plate between the dorsum of the metacarpal head and the base of the proximal phalanx converting a simple into a complex MPJ dislocation. Intrinsic tightness is a secondary rather than a primary problem. Release is not indicated for either simple or complex dislocation. The open procedures are reserved for complex dislocations.

43. A 53-year-old female with rheumatoid arthritis affecting both hands for the past 10 years complains of inability to flex her thumb for two weeks. There has been no pain or snapping. The most likely diagnosis is: 1) FPL rupture from attrition. 2) FPL rupture from direct invasion. 3) trigger thumb. 4) anterior interosseous nerve compression. 5) destruction of the MP joint. Preferred Response: 1 Discussion: Rheumatoid arthritis can and does produce proliferative tenosynovitis that directly invades and ruptures flexor or extensor tendons, but it more commonly causes bony erosions that produce an attritional rupture of the tendons. There is seldom a prodome before tendon rupture in RA, unlike a trigger thumb or finger, which usually has a period of triggering, snapping or pain before becoming locked in flexion or extension. Anterior interosseous nerve compression, a relatively rare condition, can occur in patients with RA, but the incidence would not be any greater than for the general population. 44. Which of the following statements regarding the evaluation of patients with carpal tunnel syndrome is true?

1) Routine wrist radiographs should be performed because abnormalities are seen in 33% of patients. 2) Two-point discrimination tests will be abnormal once a few fibers of the nerve demonstrate a conduction abnormally. 3) Distal sensory latency of greater than 2.5 mm/sec is consistent with carpal tunnel syndrome. 4) The carpal tunnel compression test has been found to be more sensitive and specific than Tinel's or Phalen's tests. 5) The symptoms of carpal tunnel syndrome usually have an acute onset. Preferred Response: 4 Discussion: A recent study in patients with electrodiagnostically-proven carpal tunnel syndrome shows that the carpal compression test was more sensitive and specific than either the Tinel percussion test or the Phalen wrist flexion test. Wrist radiographs are abnormal in 33% of patients with carpal tunnel syndrome but have therapeutic significance in only .4% of cases and should not be routinely ordered. Two-point discrimination remains intact even if only a few fibers are conducting normally. Distal sensory latency greater than 3.5 mm/sec is consistent with carpal tunnel syndrome. The symptoms of carpal tunnel syndrome occur gradually. 45. A 20-year-old white male jams his right long finger. Examination reveals palpable tenderness and swelling about the radial side of the PIP joint. Radiographs show I mm joint space widening on the radial side of the PIP joint, but no fractures. Lateral stress testing reveals more than 20~ angulation. Treatment should include: 1) Immobilize 10 days; then begin intermittent active motion with continued splinting. 2) Immediate range of motion; elastic wrap to control swelling. 3) Open ligament repair. 4) Transarticular K-wiring for three weeks, then active range of

motion. 5) Buddy tape to index finger and begin immediate range of motion. Preferred Response: 3 Discussion: The apex radial angulation caused by slight joint space widening on the radial side is suggestive of interposition ligament fibers and probable complete tear of the radial collateral ligament and radial side of the volar plate. Kieflhaber, et. al. reported that complete tears of PIP joint collateral ligaments will be associated with more than 20 of angulation on stress testing. The lateral collateral ligament is frequently folded into the joint and surgical repair is recommended. Although significant instability may sometimes heal satisfactorily, interposition of ligament tissue will not. 46. Normal excursion of the ulnar nerve above and below the elbow is approximately: 1) 1 cm above and 3 mm to 6 mm below the elbow 2) 3 mm to 6 mm above and 1 cm below the elbow 3) 2 cm above and 2 cm below the elbow 4) 2 cm above and 1 cm below the elbow 5) 1 cm above and 2 cm below the elbow Preferred Response: 1 Discussion: Neural excursion and elongation are obligate properties of peripheral nerve that provide for accommodation to the changes in length and geometry associated with normal motion. A spiral orientation of axonal and connective tissue within peripheral nerve may allow elongation of the nerve trunk to occur without concomitant changes in the strain of individual axons, within certain limits. The epineurium acts as an interface between the

external environment of the nerve bed and the internal components of the peripheral nerve trunk. External gliding occurs between the nerve bed, neuromesentary, and epineurium, while internal gliding appears to occur between the epineurium, interstitial connective tissue, and nerve fascicles. Apfelberg and Larson noted that neuromesentary allows I cm of human ulnar nerve excursion above the elbow and 6 mm below it. Wilgis and Murphy found 9.8 mm of the ulnar nerve excursion above and 3.2 mm of excursion below the elbow. The correct answer in this case is a range that encompassed both of these studies.

47. A 29-year-old right-handed plumber suffers a dorsal dislocation of the PIP joint of the ring finger. A congruent reduction is performed and the reduction is stable through a full ROM under anesthesia (digital block). The physician should now recommend: 1) Elevate for 36 hours, splint only the PIP joint in 5 of flexion; start motion exercise in seven to 10 days. 2) Elevate for seven days or more if swelling persists, splint only PIP joint in 45 of flexion; start motion at three weeks. 3) Elevate until acute inflammation subsides; splint the PIP joint 45 of flexion; start motion exercise at seven to 10 days. 4) Elevate until acute inflammation subside; splint the PIP joint in 5 of flexion; start motion exercise at seven to l0 days. 5) Elevate for 36 hours; buddy tape the ring to the middle finger; immediate motion exercise. Preferred Response: 4 Discussion: The most common long-term complication of stable reductions of PIP joint dorsal dislocations is stiffness and a degree of permanent swelling. Swelling will be minimized if elevation is continued until the acute inflammatory response to the injury has subsided, which usually takes at least seven to 10 days. Splinting the PIP joint in 45 of flexion for three weeks will result in volar

plate adhesions and a difficult to correct flexion contracture. With a stable reduction motion in a figure-of-eight extension block splint can be started safely once pain is minimal, usually seven told days. Buddy taping would be acceptable but with ulnar collateral ligament injury, should be taped to the small finger.

48. A 65-year-old male complains of decades of pain and marked stiffness of the fingers. His range of motion is shown in Figures 11A and 11D. X-rays of photos are shown in Figures 11E and 11F. Rheumatoid titer levels have been normal. HLA-BW17 and HLA-BW38 titers are elevated. His most likely diagnosis is: 1) psoriatic arthritis 2) gout 3) pseudogout 4) rheumatoid arthritis 5) osteoarthritis

Preferred Response: 1 Discussion: While gout or pseudogout can occur in conjunction with psoriatic arthritis, the history of decades of unremitting symptoms make this less likely. Clinical photos show pitting of the nails, characteristic of psoriatic arthritis, and X-rays show involvement of MCP, PIP, and DIP joints. The rheumatoid factor is persistently negative, which argues against rheumatoid arthritis. The marked osteolysis of the DIP joint is not seen in osteoarthritis and such bone resorption is usually associated with generalized osteopenia in rheumatoid arthritis. This combination

of osteolysis adjacent to bone of normal density and panarticular erosions, rather than marginal erosions, is characteristic of psoriatic arthritis. Finally, the marked tenosynovitis and "sausage finger" are characteristic of psoriatic arthritis.

49. The sensibility test with the highest level of inter-rater reliability is: 1) Tuning fork. 2) Two-point discrimination. 3) Ridge device. 4) Semmes-Weinstein monofilaments. 5) Ninhydrin test. Preferred Response: 4 Discussion: The level of variance using hand-held touch stimuli is inherently high. As testing instruments, monofilaments are unique in their ability to control the amount of force applied. These produce the most sensitive and reliable data of all clinical sensibility assessment instruments currently available. In using two-point discrimination, differences between the amount of force applied to the one point and that applied to the two points easily exceeded the sensory threshold for normal sensation, even among experienced surgeons and hand therapists. This lack of force becomes amplified during moving two-point discrimination. Inter-rater reliability is poor. Ridge device (sensitometer) introduces tissue displacement rather than applied force. However, validity and reliability remain questionable. Vibratory instruments (tuning fork) similarly lack control of force during instrument application, therefore affecting reliability. Also, most available vibration instruments are of fixed frequency that vary the intensity; they do not provide variable frequencies at fixed intensities. Ninhydrin test identifies areas of disturbance of sweat secretion following peripheral nerve disruption. This test is not specifically used to evaluate sensibility, it tests pseudomotor

function. 50. Lupus arthritis in the hand is characterized by: 1) Extremely stiff interphalangeal joints 2) Erosion of subchondral bone and joint space narrowing 3) "Punched-out" bone lesions in the peri-articular areas 4) Deformities of the digits without joint destruction 5) Frequent involvement of the distal interphalangeal joints and nail pitting Preferred Response: 4 Discussion: Systemic lupus erythematosus is more prevalent in young women and in the hand causes joint deformities as the result of ligament and volar plate laxity and tendon subluxation that leads to joint imbalance without the erosive destruction of articular cartilage seen in other forms of arthritis. Psoriatic arthritis results in extremely stiff joints frequently involving the DIP joints and is commonly accompanied by nail pitting. Rheumatoid arthritis causes bone erosion and joint space narrowing, while gout can cause circumscribed bone erosion or peri-articular "punched-out" lesions.

51. Which of the following is the most accurate description of the local tissue response to a tissue expander? 1) Epidermis thins. 2) Dermis thins. 3) Subcutaneous tissue hypertrophies. 4) Adjacent muscle enlarges. 5) Local vascularity decreases. Preferred Response: 2 Discussion:

The dermis shows a decrease in thickness following insertion of a tissue expander. This thinning is not related to the volume of expansion, the anatomic location, or the rate expansion. The collagen bundles are noted in a characteristic orderly and parallel alignment. The epidermis initially increases in thickness. This initial thickening will gradually return to original baseline values with further expansion. Tissue expansion causes significant atrophy of muscle, as well as thinning of the subcutaneous adipose tissue. Expanded tissue shows an increase in vascularity. 52. A 64-year-old woman presents with a four-month history of snapping and locking of the long finger with difficulty extending the finger at the MP joint. With flexion, the extensor tendon shifts ulnar to the MP joint and the finger locks in flexion and slight ulnar deviation at the MP joint. Once the finger is passively extended, alignment is restored and she can then actively maintain the extended posture. Treatment should consist of: 1) Injection of the flexor tendon sheath. 2) Release of the ulnar intrinsic tendon. 3) Reconstruction of the radial sagittal band. 4) Release of the Al pulley. 5) Release of the ulnar collateral ligament. Preferred Response: 3 Discussion: Rupture of the radial sagittal band results in ulnar subluxation of the extensor mechanism at the MP Joint with snapping and the posture noted in this patient. If seen acutely, this injury is treated by splinting the MP joint in extension. When seen late, it is best managed by reconstruction of the radial sagittal band. The findings are distinctly different from those seen in trigger fingers, in which the sagittal alignment of the finger is not altered and the interphalangeal joint locks, as opposed to the MP joint. The UCL and ulnar intrinsic do not play a role in the pathology of this entity.

53. Which of the following statements regarding isolated avulsion of the flexor digitorum superficialis is true? 1) Isolated rupture of the FDS is more common than isolated rupture of the flexor digitorum profundus. 2) Isolated flexion of the digit reveals a mass at the A4 pulley. 3) Most patients display limited active proximal interphalangeal joint motion at the time of diagnosis. 4) Proximal migration of the ruptured tendon ends are invariably found at the level of the proximal carpal canal. 5) Rheumatoid arthritis, tenosynovitis, fractures, and tuberculosis are not related to closed tendon avulsion of FDS or FDP. Preferred Response: 3 Discussion: A recent series of 11 patients with isolated, closed avulsions of the FDS tendon reported on physical findings and the effect of treatment. All were found to have a tender palmar mass at the Al pulley and loss of active FDS function. In addition, all had some loss of PIP ROM, many with a PIP contracture requiring surgical release. Surgical exploration revealed the retracted FDS ensnared around FDP tendon at the Al pulley. Following tenolysis, FDS excision and capsulotomy, when indicated, each patient achieved a functional result. Rheumatoid arthritis, tenosynovitis, fractures, bony abnormalities and tuberculosis have all been implicated in closed tendon rupture of the FDS or FDP. 54. A 42-year-old female cut her index finger with a clean knife over the volar aspect of the mid proximal phalanx. She sought no medical attention at the time of the injury. Subsequently, she noted a painful clicking of this finger with flexion and extension. Her active and passive range of motion is normal. She is referred to you after treatment by another surgeon that included a cortisone injection first and then release of the Al pulley. Treatment should consist of:

1) observation. 2) another cortisone injection. 3) debridement of the partial tendon laceration(s). 4) incision of the A2 pulley. 5) excision of the radial slip of the sublimis tendon. Preferred Response: 3 Discussion: This patient has a mechanical problem from a partial laceration of a flexor tendon that is catching on the tendon sheath. This problem is not caused by inflammation and therefore will not respond to an injection. Incision of the A2 pulley will most likely cause bow stringing. If the injured tendon is debrided without injury to additional annular pulleys the clicking and pain should resolve. Excising the sublimis tendon is not necessary.

55. A young adult sustains the closed injury shown in Figures 12A and 12B. Treatment should consist of: 1) External fixation 2) Minicondylar plate 3) Two Kirschner wires 4) A single Kirschner wire 5) Dynamic splinting

Preferred Response: 3

Discussion: Unicondylar fractures require fixation to assure healing without displacement. A single Kirschner wire will not prevent fragment rotation while a second Kirschner wire or two mini screws will. Neither dynamic splinting nor external fixation will reliably control the fracture fragment. Miniplates are not indicated for this fracture.

56. The sympathetic nerve fibers to the ulnar artery at the level of Guyon's canal are typically derived from: 1) The deep motor branch of the ulnar nerve. 2) The nerve of Henle'. 3) The Riche' - Cannieu anastomosis in 25% of cases. 4) Fibers from the nerve to the opponens pollicis. 5) Fibers from the common digital nerve to the fourth web space. Preferred Response: 2 Discussion: The nerve of Henle' first described in 1868, was studied by McCabe and Kleinert in 1989. This branch of the ulnar nerve may arise in the mid or distal forearm and typically gives off fibers to the ulnar artery. These are thought to carry sympathetic innervation to the ulnar artery and are of importance when performing local svmDathectomy with excision for ulnar artery thrombosis. 57. A 45-year-old mechanic is referred because of continued pain in the volar proximal forearm and numbness in the thumb, index, long, and ring fingers, and thenar eminence despite a carpal tunnel release six months ago. The symptoms are exacerbated with active tool use, and are absent at night. Tinel's and Phalen's signs are negative at the wrist. Percussion over the proximal volar forearm is painful with paresthesias radiating to the fingers. The

most likely diagnosis is: 1) Incomplete release of the transverse carpal ligament. 2) Anterior interosseous syndrome. 3) Thoracic outlet syndrome. 4) Wartenberg's syndrome. 5) Pronator syndrome. Preferred Response: 5 Discussion: The predominance of forearm pain, absence of night symptoms, and thenar numbness are atypical for carpal tunnel syndrome and more consistent with pronator syndrome, hence the absence of relief with carpal tunnel release. The anterior interosseous nerve is a purely motor branch of the median nerve. Compression causes weakness of the protundi to the index and long fingers and FPL, but no sensory symptoms. Thoracic outlet syndrome usually presents with lower trunk/medial cord (ulnar nerve distribution) symptoms. Wartenberg's syndrome refers to compression of the superficial radial nerve at the distal forearm.

58. A 22-year-old male jams his middle finger playing basketball. Examination reveals that the DIP joint rests in 30 of flexion and that the patient is unable to actively extend the joint from this position. Radiographs reveal a fracture of the dorsal lip of the distal phalanx involving 20% of the articular surface. There is no volar subluxation of the distal phalanx. Treatment should consist of: 1) Static splinting; DIPJ in flexion; PIPJ in extension 2) Miniscrew fixation of the fragments 3) Static splinting; DIPJ in extension; PIPJ in extension 4) Tension-band wire fixation of the fragments 5) Static splinting; DIPJ in extension; PIPJ free Preferred Response: 5

Discussion: Static splinting of the DIPJ in extension and the PIPJ free is indicated for bony mallet finger deformities with small fracture fragments and no subluxation. Splinting the DIPJ in flexion would accentuate the displacement of the dorsal fragment. Splinting the PIPJ in extension would risk permanent stiffness of that joint. Screw or tension-band wiring would have a substantial risk of shattering such a small fragment and may contribute to DIP joint stiffness. 59. Which of the following statements is true regarding ulnar club hand? 1) It is frequently associated with cardiac, hematopoietic, and gastrointestinal problems. 2) It is rarely associated with syndactyly. 3) It is frequently associated with elbow abnormalities. 4) Surgical treatment consists of early wrist fusion. 5) It is associated with a high familial incidence. Preferred Response: 3 Discussion: Ulnar club hand is far less common than radial club hand. It is more commonly associated with other musculoskeletal anomalies and not visceral abnormalities. Although rare familial cases have been reported, this occurrence is felt to be sporadic. Syndactyly is present in 34% of patients. In radial club hand, the elbow may have an associated contracture but is mechanically stable. Ulnar club hand, however, is frequently associated with elbow instability and possibly radial head dislocation or radiohumeral synostosis. The wrist is stable and does not require fusion. Treatment varies and maybe directed towards resection of the ulna anlage, and radial or humeral osteotomies.

60. Severe swelling of the dorsum of the hand following injury to the hand and wrist may produce stiffness, even of uninjured parts because: 1) it is so painful, the patient cannot do exercises. 2) when the dorsal skin is expanded by swelling it will splint the MCP joints in extension. 3) it may lead to infection. 4) RSD will occur next. 5) it will cause arthritis of the small joints of the hand. Preferred Response: 2 Discussion: Excessive swelling of the dorsum of the hand may or may not be accompanied by pain but will always splint the MCP joints in extension with the collateral ligaments in the shortened position. This frequently leads to a permanent extension contracture. RSD is usually preceded by swelling but its cause is multifactorial and swelling alone may not lead to it. In the absence of an open wound, swelling is not likely to be a proximate cause of infection. Swelling may cause arthrofibrosis of small joints but not arthritis. 61. A 45-year-old female complains of thenar eminence numbness and severe dysesthesia six months following a carpal tunnel release. Exam reveals a well healed scar that obliquely crosses the wrist flexion crease between the palmaris longus and flexor carpi radialis tendons, with marked sensitivity, Conservative care has failed to relieve her symptoms. Treatment should consist of: 1) Release of the flexor carpi radialis tendon sheath. 2) Scar excision with full-thickness skin graft. 3) Repeat carpal tunnel release. 4) Transposition of palmar cutaneous neuroma. 5) Repair of the recurrent motor branch of the median nerve. Preferred Response: 4

Discussion: The palmar cutaneous branch of the median nerve provides sensation to the thenar eminence and emerges proximal to the wrist flexion crease between the palmaris longus and the flexor carpi radialis tendons. Knowledge of its anatomy is essential to avoid injury and subsequent neuroma tormation. In the event of symptomatic neuroma formation, transposition can provide some improvement. The other choices do not address the source of the patient's problem.

62. A four-year-old girl accidentally lacerates her palm. Two weeks after wound closure she complains of increasing pain and swelling about the wound. Examination reveals a tender pulsatile mass. The clinical appearance of her hand is shown in Figure 13A and the appearance of the mass at surgical exploration in Figure 13B. The most likely diagnosis is: 1) Wound hematoma. 2) Mycotic aneurysm. 3) False aneurysm. 4) Ateriovenous malformation. 5) True aneurysm.

Preferred Response: 3 Discussion:

This is a pseudo-aneurysm that occurred from iatrogenic injury to the ulnar artery at the time of suture. Unlike true aneurysms, these contain no arterial wall elements. Mycotic aneurysms are seen most commonly in IV drug abusers. Pseudo-aneurysms can occur in hemophiliacs, but this is an x-linked recessive disease occurring in males. 63. A 36-year-old right-handed female has a nine-month history of deQuervain's stenosing tenosynovitis. She has failed conservative and surgical treatment is planned. A complication unique to the recommended procedure is: 1) volar tendon subluxation. 2) infection. 3) swelling and stiffness. 4) neuroma of the radial sensory nerve. 5) Prieser's disease. Preferred Response: 1 Discussion: The patients diagnosis is deQuervain's disease and the proper surgical solution is incision of the first extensor tendon compartment. Choices (2) and (3) are possible with any operation. While neuroma of the radial sensory nerve may occur with this surgery, it is a risk as with any surgery in this area, i.e. basal joint arthroplasty, triscaphe arthrodesis. Volar subluxation of the tendons of this compartment can occur even if the sheath is cut on its most ulnar border and, when symptomatic, may require pulley reconstruction. Prieser's disease is idiopathic avascular necrosis of the scaphoid.

64. A 67-year-old male returns home after a golf and fishing trip to Florida. He complains of severe pain and swelling in his left, non-dominant wrist. There is no history of trauma or systemic complaints. Radiographs show a mild scapholunate dissociation

and calcific stippling in the TFCC region. Treatment should consist of: 1) immediate irrigation and debridement of the wrist joint. 2) splinting and anti-inflammatory medication. 3) STT fusion. 4) aspiration and culture for atypical mycobacteria. 5) oral antibiotics. Preferred Response: 2 Discussion: The most likely diagnosis is an acute pseudogout attack with deposition of calcium pyrophosphate within the TFCC. Infection, either with bacteria or mycobacterium marinum is within the differential diagnosis but are very rare in comparison with pseudogout, which commonly affects the wrist in this age group of patients. 65. An 18-year-old man sustains an accidental gun shot wound to the distal forearm. His injuries include an open, segmental radius fracture and laceration of the median nerve. Following debridement of the wound and stabilization of the radius fracture, he is left with a 6 cm defect in his median nerve. Which of the following is true regarding treatment of the nerve defect? 1) The sural nerve can provide up to 20 cm of useful nerve graft material. 2) Vascularized nerve grafting produces superior results compared to non-vascularized grafting. 3) Aligning the motor and sensory fascicles in the proximal and distal stumps is an important factor in good results. 4) Using sensory re-education, S3+ sensory function has been reported in only 30% of patients. 5) Because of the length of defect, primary repair or grafting should not be done and tendon transfers should be done at a later time.

Preferred Response: 3 Discussion: Nerve defects up to 8 cm or 10 cm in length are amenable to nerve grafting. The sural nerve can provide up to 40 cm of nerve graft material. Both experimental and clinical studies on vascularized nerve grafting fails to show any advantage over non-vascularized grafting. The ability to align proximal and distal nerve stumps in their correct orientation prior to grafting dramatically improves results. S3+ sensory function has been reported in 80% of patients following median nerve grafting.

66. An axial flag flap raised on the dorsum of the long finger utilizing the radial dorsal digital artery can be used to cover a defect on: 1) The volar distal palm at the based of the ring finger. 2) The dorsal proximal interphalangeal joint of the long finger. 3) The ring-long web space. 4) The index volar pad. 5) The volar distal palm at the base of the index finger. Preferred Response: 5 Discussion: The axial flag flap utilizes the arterial supply of the dorsal digital artery. It is most reliably taken from the dorsal aspect of the proximal phalanx of the index or long fingers. When it is taken from the long finger and based radially using the radial dorsal digital artery, it can be used to cover defects over the dorsal MP joint of the long or index finger or the dorsal base of the index finger. It can also be used to cover the volar aspect of the base of the long or index finger or the distal palm at the base of the long and index fingers. It can reach defects on the palmar surface of the MP joint of the index or long fingers by carrying the flap through the web space. Because the described flap was based

radially, it cannot be used to reach the ring-long finger web or the volar distal palm at the base of the ring finger. Since it is based proximally, it cannot reach the dorsal proximal interphalangeal joint of the long finger. Based on the lack of flexion of the index finger, it would be virtually impossible to use it to cover the index finger volar pad. 67. A 70-year-old female is initially treated for a non-displaced distal radius fracture in a short arm cast that extends to the palmar digital crease shown in Figures 14A and 14B for six weeks. After concerted and exhaustive rehabilitation elforts, her range of motion is seen in Figures 14C and 14D. The patient reports that this is a serious functional limitation. Recommended treatment is: 1) dynamic splinting 2) MCP arthroplasties 3) metacarpal neck osteotomies 4) extensor tenolysis 5) dorsal capsulotomy and collateral ligament resection

Preferred Response: 5 Discussion: The initial cast extended too far distally. The level of the MCP joint corresponds to the distal palmar crease, not the palmar digital crease. This has allowed the collateral ligaments to tighten in extension and prevents flexion of the MCP joints. This is a common and serious problem that is frequently not responsive to even aggressive therapy. MCP joint arthroplasty is not required because there is nothing wrong with the articular surfaces. Metacarpal neck osteotomy would produce little gain in motion

because the ligamentous and capsular contractions are not released. Thus, the best option would be dorsal capsulotomy and release or recession of the collateral ligaments. 68. A medical student fell on his hands while roller blading five years ago and complains of bilateral, severe ulnarsided wrist pain. X-rays are shown in Figures 15A and 15B. Conservative measures have been exhausted and recommended treatment is: 1) TFCC repair on the left and excision of the styloid fragment on the right 2) bilateral ulnar styloidectomy 3) left ulnar shortening diaphyseal osteotomy and right arthroscopic TFCC repair 4) "wafer" procedure on the left and styloid fragment excision on the right 5) Darrach procedure and ECU/FCU tenodesis stabilization on the left and right styloid fragment excision

Preferred Response: 1

Discussion: While ulnar styloid fracture nonunions are common, symptoms and infrequent and may be due to the fibrous nonunion or concomitant TFCC injury. More recently, Hauck, et. al. report on the classification and treatment of symptomatic ulnar styloid nonunion. Injuries associated with DRUJ instability were treated by TFCC reconstruction, while nonunions with no instability underwent simple excision of the fragment. This patient's radiographs demonstrate an ulnar styloid nonunion on the right side with a stable DRUJ (Type 1). On the left side, there appears to be an old, healed styloid fracture, but there is residual DRUJ instability seen in the stress AP and lateral films. The correct treatment would be left TFCC repair and excision of the fragment on the right. 69. In fractures of the distal radius involving younger adults, the most important prognostic indicator for long-term successful results is the restoration of: 1) Radial length (<3 mm shortening) 2) Articular joint congruity (<2 mm step-off) 3) Radial angulation (<15 dorsal tilt) 4) Radial shift (<3 mm) 5) Radial inclination (>10) Preferred Response: 2 Discussion: Symptomatic arthritis has been shown to develop in 90% of younger adults who healed with residual articular surface step-offs of 2 mm or greater whereas it occurred in only 11 % of those that healed with a congruous surface. Radial length, angulation (tilt), inclination and shift also influence outcomes in decreasing order of importance.

70. A 43-year-old physician sustains a non-displaced fracture of the distal radius while skating. She is placed in a shortarm cast. Five weeks after injury she notes an inability to move her thumb. Treatment should consist of: 1) Corticosteroid injection at the Al pulley. 2) Short-arm spica cast. 3) IP joint arthrodesis. 4) Primary repair of the EPL. 5) EIP to EPL transfer. Preferred Response: 5 Discussion: A trigger thumb can cause an inability to move the thumb and can occur during immobilization but it is usually associated with volar thumb discomfort. Other possibilities in the differential diagnosis include compression of the posterior interosseous nerve, but the most common possibility is rupture of the extensor pollicus longus. A primary repair of the EPL is seldom possible after the attritional rupture of this tendon after distal radius fracture. Therefore, a reconstruction is usually the only option and can include a free tendon graft or EIP to EPL transfer. 71. The parents of a three month-old infant bring the child to your office for evaluation of the right thumb. They have noticed that he holds the digit flexed at the IP joint, and has done so for the past month. On examination, the IP joint extends to 30 passively, with a mechanical block to further extension. The remainder of the hand examination is normal. Recommended treatment is: 1) EIP to EPL tendon transfer. 2) Operative release of the Al pulley of the thumb within one month. 3) Echocardiogram and IVP. 4) Operative release of the Al pulley at the age of one year if the deformity is still present.

5) Full-time splinting for six months. Preferred Response: 4 Discussion: The etiology of the congenital trigger thumb is unknown. It occurs as an isolated lesion and is not associated with other visceral anomalies. While the problem is called "congenital," not all children are affected at birth. Some authors feel that 30% of trigger thumbs resolve by the age of one year. Consequently, most recommend observation of the lesion until one year of age when anesthesia risk is lower and thumb structures are larger. Splinting has not been shown to be beneficial. Surgical release of the Al pulley is curative and can be performed after three years of age with no residual deformity according to Skov, Bach, and Hammer. 72. A 30-year-old cook lacerates the back of his hand with a large butcher knife. The oblique direction of the cut produces an extensor Zone IV laceration of the index finger, Zone V laceration of the long finger, and a Zone VI laceration of the ring finger. A11 tendons are repaired without difficulty. Postoperatively, which finger(s) has the highest likelihood of a good or excellent result? 1) index 2) long 3) ring 4) index and long 5) long and ring Preferred Response: 2 Discussion: The distal Zones (I-IV) do significantly more poorly than the more proximal zones. Zone IV has a 43% good or excellent result rate. Zone V, even with capsule laceration, has an 83%

good or excellent result rate, and Zone VI has a 65% good or excellent result rate. 73. In a patient with rheumatoid arthritis who has a stable, well balanced and painless wrist, painful subluxated MP joints of the fingers, and extensor tendon ruptures, the correct sequence of a staged reconstruction is: 1) MP joint reconstruction then extensor tendon reconstruction 2) Extensor tendon reconstruction, then MP joint reconstruction 3) MP joint reconstruction, then wrist fusion 4) Extensor tendon reconstruction, then MP joint fusion 5) Wrist arthroplasty, then extensor tendon reconstruction Preferred Response: 1 Discussion: In a rheumatoid patient with MP joint destruction requiring arthroplasty who also has extensor tendon ruptures, staged reconstructive surgery often yields better results. The MP joint surgery should be done first. Motion of the reconstructed joints can be maintained with dynamic splinting until sufficient healing has occurred to allow proceeding with extensor tendon reconstruction as the second stage. If the extensor tendon surgery is done before the joint arthroplasties, the reconstructed tendons will be unable to move the subluxated and arthritic joints through the range of motion necessary to maintain tendon function. 74. A 52-year-old professional string bass player fell from a ladder and suffered a bilateral wrist injury. He complains of localized pain on the dorso-ulnar aspect of his right wrist (Figure 16A), his bow hand. His left wrist (Figure 16B) is swollen and generally painful. He wishes to return to practicing and performing as soon as possible. Recommended treatment is: 1) ORIF right wrist and ORIF left wrist (plate and screws) . 2) SA cast right wrist and external fixation left wrist. 3) SA cast right wrist and closed reduction and SA cast left wrist.

4) SA cast right wrist and ORIF left wrist (plate and screws). 5) SA cast right wrist and ORIF left wrist (intrafocal pins).

Preferred Response: 4 Discussion: This patient has two special requirements early return to performance and enough function in his left wrist to allow proper hand position for fingering his instrument. His right bow hand can function with a short arm-cast which is the preferred treatment for avulsion fracture of the triquetrum. To assure that he can have adequate wrist flexion and early return to function, anatomic reduction and rigid internal fixation is required on the right. This can best be accomplished with either a dorsal or volar blade plate.

75. A 20-year-old student falls while cross country skiing. His thumb is shown in Figure 17A and X-rays are shown in Figures 17B and 17C. Treatment should include: 1) splinting until comfortable and early rehabilitation permitting use as tolerated 2) surgical repair of the dorso-radial capsule and EPB insertion 3) surgical repair of the radial collateral ligament 4) repair and advancement of the APB insertion 5) repair of the radial collateral ligament and dorsoradial capsule

Preferred Response: 5 Discussion: The clinical photo shows a bulge on the radial aspect of the thumb MCP joint and ulnar angulation of the thumb at the MCP joint with the thumb in repose. This is due to loss of the primary radial stabilizer of the joint, the radial collateral ligament complex. X-rays shown in Figure 17B and 17C confirm the diagnosis of radial instability and show palmar subluxation of the proximal phalanx on the metacarpal head. While injuries to the ulnar collateral ligament are more common than radial-sided injuries, the structure and importance of the radial complex is receiving increasing attention. The role of the dorsal capsule and EPB insertion, as well as the RCL, in supporting the proximal phalanx and preventing palmar subluxation of the proximal phalanx has been described by Posner and Retaillard, among others. Failla recently emphasized the importance of repairing the injured structures and confirmed that the injury may involve the RCL, EPB/capsule or both. Given the degree of instability demonstrated in this patient, surgical repair is indicated. The marked radial instability and palmar subluxation reflect injury to both the RCL and EPB-dorsal capsule complex.

76. A patient sustains a closed, complex dorsal dislocation of his index MCP joint (Figures 18A and 18B). The anatomic structure in greatest jeopardy of iatrogenic operative injury during a volar surgical approach to correct the problem is: 1) Lumbrical muscle. 2) Ulnar digital nerve. 3) Flexor digitorum superficialis. 4) Radial digital nerve. 5) Flexor digitorum profundus.

Preferred Response: 4 Discussion: The radial digital nerve is tented over the region of the Al pulley. The Al pulley must be incised during a volar approach in order to extract the volar plate and reduce the dislocation. It is just beneath the skin and is the structure most likely to be injured during this procedure. A dorsal approach does not place this structure at risk for injury.

77. After nerve laceration, Wallerian degeneration: 1) occurs in the proximal stump. 2) occurs by degeneration of Schwann cells. 3) causes no change within the cell body.

4) clears degraded myelin and axoplasm from Schwann cell tubes. 5) causes immediate lack of response in the distal segment during nerve conduction studies. Preferred Response: 4 Discussion: Wallerian degeneration occurs within the distal segment of the nerve after laceration. The Schwann cells and the tube they form remains intact and untouched. The contents of the Schwann cell tube undergo granular degeneration and are ultimately cleared by macrophages. The cell body remains intact but increases production of growth associated protein 100 fold in response to the injury. Until complete degeneration in the distal axons occurs, there is still capacity for electrical conduction for up to three weeks after injury. 78. Which of the following statements is true regarding MRI for Zone 11 flexor tendon injuries? 1) MRI clearly differentiates tendon ruptures from adhesions with 100% accuracy. 2) MRI cannot clearly differentiate adhesions from tendon rupture. 3) Clinical examination for tendon rupture or adhesion is more accurate than MRI evaluation. 4) Ultrasonography more reliably evaluates partial tendon ruptures in Zone II. 5) MRI delineates soft-tissue structures in the hand poorly. Preferred Response: 1 Discussion: Differentiation of tendon rupture from tendon adhesion may be a difficult clinical problem and accurate diagnosis is critical. MRI has been shown to accurately differentiate tendon rupture from

tendon adhesion. MRI has been shown to be more accurate than clinical examination and more accurate than ultrasonography. 79. A patient develops painful snapping of the exterior tendon over the metacarpophalangeal joint of the middle finger following an industrial accident two months ago. The patient is now unable to fully extend at the metacarpophalangeal joint and ulnarly deviates with active extension. The most appropriate management of this injury includes: 1) Static splint with MCP joint in extension for four weeks; immediate mobilization of the PIP/DIP. 2) Static splint with MCP joint in 70^ flexion for four weeks; immediate mobilization of the PIP/DIP joints . 3) Cast immobilization with hand in a "safe" position for four weeks. 4) Repair of the sagittal band and stabilization of the extensor tendon with tendon graft. 5) Dynamic MCP joint extension splint. Preferred Response: 4 Discussion: The patient has developed a chronic dislocation of the extensor tendon over the metacarpophalangeal joint secondary to an injury to the radial sagittal band. Acute injuries seen within two weeks should be treated with immobilization preventing flexion at the metacarpophalangeal joint but allowing IP joint motion. Chronic injuries more than two weeks old require surgical repair of the defect and stabilization of the extensor tendon. 80. A 75-year-old male with severe spastic hemiplegia two years following a stroke, has skin maceration of his fingers and palm on the involved side, secondary to severe flexion deformities of all fingers. Local skin hygiene is a major problem. Recommended treatment is: 1) Phenol injection.

2) Intrinsic tendon releases. 3) MP joint arthrodesis in extension. 4) Superficialis to profundus tendon release and transfer. 5) Transfer of the FCU to the finger extensors. Preferred Response: 4 Discussion: Spasticity following a stroke can sometimes be so severe that the fingers dig into the palm, macerating the skin and hindering hygiene. The spastic muscles are the extrinsic finger flexors, hence intrinsic releases would not solve the problem. Transfer of the FCU to the finger extensors would not release or overcome the spastic finger flexors. Phenol blocks can be helpful for temporary relief of spasticity and may be used while healing or training a tendon transfer, but would not be a permanent solution. Finger arthrodesis would be more complicated than necessary and MCP joint arthrodesis would not correct flexion deformities at the IP joints. Superficialis to profundus transfer effectively lengthens the spastic extrinsic finger flexor tendons, and still allows the patient to retain some finger control. 81. A 23-year-old roofer presents to an emergency room two hours after falling on an outstretched wrist. There were no other injuries and the distal neurosensory exam is normal. Radiographs are depicted in Figures 19A and 19B. Treatment should include: 1) closed manipulation and long-arm cast immobilization 2) scaphoid internal fixation and internal/external fixation radius 3) dorsal plate fixation of the radius and short-arm thumb spica 4) bone graft and internal fixation of the scaphoid plus long-arm cast 5) percutaneous pins and bone graft to radius plus bone graft to scaphoid

Preferred Response: 2 Discussion: Concomitant fractures of the distal radius and scaphoid are uncommon. Most distal radius fractures require some flexion if immobilized in a cast; a position not recommended for scaphoid fractures particularly if there is comminution of the anterior scaphoid cortex. Therefore this injury requires operative management. Recent literature suggests that the scaphoid requires internal fixation either with pins or a screw. Management of the radius fracture must be individualized and depends on the fracture pattern, degree of combination, and bone compaction. Proubasta and Lluch recommend external fixation alone. This could be risky because the scaphoid could be distracted and distal radius may not completely reduce. 82. Which of the following is a contraindication to transfer of the flexor carpi ulnaris tendon for wrist extension in a patient with cerebral palsy? 1) Intelligence quotient of 75. 2) Pure athetoid pattern. 3) Hand-to-knee placement of less than five seconds. 4) Two-point discrimination of 8 mm. 5) Pre-op EMG showing FCU active in grasp only. Preferred Response: 2

Discussion: Tendon transfers in patients with athetoid cerebral palsy are contraindicated. Tendon transfer in the athetoid patient may actually worsen the pre-op state. Two-point discrimination of 8 mm indicates sufficient sensibility for good outcome. IQ of 75 and hand-to-knee placement of less than five seconds are also predictors of good results. A muscle that is active in grasp only is in the appropriate phase for transfer to the wrist extensors.

83. A healthy 15-year-old hockey player presents to the office six weeks after injuring his right dominant hand in a skating accident. He struck his hand against the boards and complained of an occasional "snapping" sensation with intermittent difficulty in extending his middle finger. X-rays are unremarkable. On examination, a defect is noted dorsally over the metacarpophalangeal joint on flexion. The most likely diagnosis is: 1) Traumatic dislocation of the extensor tendon associated with rupture of the sagittal bands. 2) Traumatic rupture of the extensor digitorum communis. 3) Posttraumatic flexor tenosynovitis with locking. 4) Stenosing tenosynovitis associated with collateral ligament instability. 5) Rupture of the junctura tendinae. Preferred Response: 1 Discussion: Subluxation or dislocation of the extensor digitorum communis may occur following forceful flexion or extension injuries to the fingers. Ulnar dislocation is associated with a tear to the radial sagittal band and oblique fibers of the extensor hood. A traumatic rupture would eliminate any extension or possibility of "snapping." Posttraumatic synovitis and stenosing tenosynovitis would not be associated with a dorsal defect which occurs when

the extensor digitorum subluxate ulnarly. Rupture of the junctura tendinae would not cause intermittent symptoms. 84. A 57-year-old transcriptionist complains of three months of stiffness of the right ring finger when she tries to make a fist. It is worse in the morning. She also notes a burning discomfort across her distal palm. She denies injury. On examination, you find that she actively flexes to within 3 cm of the distal palmar crease when she tries to make a fist, but you can achieve full passive flexion without difficulty. The most likely diagnosis is: 1) PIP joint osteoarthritis. 2) PIP joint rheumatoid arthritis. 3) MCP joint osteoarthritis. 4) tenosynovial thickening. 5) overuse syndrome. Preferred Response: 4 Discussion: The key here is the discrepancy between active and passive flexion that points to the muscular tendinous system as the source of pathology. If the pathology is in the joint the loss of motion will be the same passively and actively. Overuse syndrome would be less likely to involve one digit and should be better with rest. Stenosing tenosynovitis is obvious if there is a triggering phenomenon. Frequently, however, it may present as morning stiffness with palmar burning pain without triggering. There will be tenderness directly over the A1 pulley. A cortisone injection can be both diagnostic and therapeutic.

85. The most common cause of a pseudoboutonierre deformity is: 1) rheumatoid arthritis 2) a central slip injury 3) psoriatic arthritis

4) a flexor tendon injury 5) a volar plate injury Preferred Response: 5 Discussion: The pseudoboutonierre deformity refers to a flexion deformity of the PIP joint that looks similar to a boutonniere deformity, but is caused by a volar plate injury, usually as the result of an acute hyperextension injury that is either untreated or treated by splinting with the PIP joint in flexion. It is differentiated from a boutonniere deformity by a history of an acute or semi-acute injury and by the absence of DIP joint extension contracture. Treatment requires splinting to stretch the contracted volar plate or surgical release of the contracted volar structures.

86. An embolus lodging in the bifurcation of the brachial artery usually originates from: 1) Subclavian artery. 2) Common iliac vein. 3) Pulmonary artery. 4) The heart. 5) Axillary artery. Preferred Response: 4 Discussion: Fifteen percent to 20% of all arterial emboli are in the upper extremity. Of these, 70% are of cardiac origin. Cardiac emboli arise from a mural thrombus that forms either after a myocardial infarction or within a ventricular aneurysm. Emboli can also be shed from mural thrombi in the atrium in association with atrial fibrillation. Emboli of cardiac origin are often quite large, and therefore lodge proximally, often in the bifurcation of the brachial artery.

87. Outcome scores for severe fractures of the distal radius treated by external fixation, which are based on function, pain, motion, and grip strength are most adversely affected by: 1) Fixator pin site bone infection 2) Duration of external fixator use for less than seven weeks 3) Overdistraction (carpal height index > 0.7) 4) Residual dorsal angulation less than 15 5) Supplemental percutaneous wire fixation Preferred Response: 3 Discussion: Overdistraction of the carpus (CHI>0.7) and prolonged duration of use of the external fixator (more than eight weeks) are associated with decreased wrist motion, grip strength and ADLs. Pin site infection, residual dorsal angulation less than 15and use of supplemental percutaneous wire fixation at the fracture site do not affect outcome scores.

88. Under digital block anesthesia, it may be possible to reduce a palmar PIPJ rotatory subluxation/dislocation by applying longitudinal traction with a gentle twisting maneuver to the middle phalanx with the hand in which position: 1) Wrist flexion 30; MP joint flexion 90; PIP joint extension 2) Wrist extension 30; MP joint flexion 30; PIP joint flexion 60 3) Wrist flexion 60; MP joint extension; PIP joint extension 4) Wrist extension 30; MP joint flexion 90; PIP joint flexion 90 5) Wrist flexion 30; MP joint extension; PIP joint flexion 30 Preferred Response: 4 Discussion:

A palmar PIP joint rotatory subluxation/ dislocation results from the buttonholing of one condyle of the proximal phalanx through a longitudinal rent between the central slip and lateral band. Flexing both MP joint and PIP joint to 90 maximally relaxes the volarly displaced lateral band (wrist extension further relaxes the extensor mechanism) allowing the entrapped lateral band to disengage and slip dorsally and the PIP joint to reduce when gentle traction with a rotational maneuver is applied . 89. A 45-year-old female falls on her dominant arm sustaining the injury shown in the X-rays in Figures 20A and 20B. A clinical photo of the medial aspect of the forearm at 24 hours after injury is shown in Figure 20C. A valgus stress view is shown in Figure 20D. Treatment should now consist of: 1) medial collateral ligament reconstruction or repair, ORIF of the radial head fracture 2) ORIF radial head fracture 3) radial head implant 4) closed reduction and casting 5) fasciotomy and subcutaneous ulnar nerve transposition

Preferred Response: 1 Discussion: The X-ray shows a comminuted radial head fracture. Stress views demonstrate medial opening of the joint with valgus stress and the clinical photo shows medial ecchymosis that indicates a serious medial injury, rather than an isolated radial head fracture. Correct treatment involves not only addressing the radial head

injury, but also the ulnar collateral ligament, which is the primary medial stabilizer of the elbow. Davidson, et. al. reported concomitant injury to either the UCL or interosseous membrane in all patients with a comminuted radial head fracture. Failure to treat the medial side risks persistent medial instability, pain and early degenerative changes. ORIF or replacement of the radial head with a silastic or titanium implant confers some additional stability, but cannot substitute for a competent UCL. The most pernicious combination is radial head excision with UCL injury. The incidence of instability, pain and tardy ulnar neuritis is quite high in this scenario.

90. Which of the following surgical procedures is not commonly performed for spastic thumb-inpalm deformity. 1) Flexor pollicis longus z-lengthening or fractional recession. 2) Thumb CMC joint arthrodesis. 3) Recession of the thenar origin. 4) Adductor pollicis origin recession. 5) First dorsal interosseous release Preferred Response: 2 Discussion: All of the procedures listed above have been described as primary or secondary procedures for the correction of spastic thumb-in-palm deformity except for CMC joint arthrodesis. While CMC arthrodesis may place the thumb in a position of desired anteposition out of the plane of the palm, it has not been widely used for the treatment of spastic thumb-in-palm deformity.

91. A 68-year-old female with rheumatoid arthritis has an adduction contracture of the first web space. Treatment should include:

1) Release of the adductor pollicis muscle from the third metacarpal 2) Z-lengthening of the flexor pollicis longus tendon 3) CMC resection arthroplasty and Z-plasty of the web skin 4) MP joint fusion and Z-plasty of the web skin 5) Web release and dorsal transposition flap Preferred Response: 3 Discussion: First web space contractures in rheumatoid patients are most commonly the result of progressive arthritis of the thumb CMC joints that results in a flexion and adduction deformity of the first metacarpal and a secondary contracture of the web space. Correction of the fixed metacarpal deformity by CMC joint arthroplasty combined with a Z-plasty of the first web skin usually are sufficient to correct the web contracture, although occasionally release of the adductor aponeurosis is required.

92. Studies on the effect of the flexor tendon pulley system show that: 1) A4 pulley excision provides the largest elficiency change. 2) A I or A5 pulley resectioning had a larger effect than A4 excision. 3) A2 pulley excision produces the largest efficiency change. 4) Pulleys improve angular motion of excursion by increasing the momentum. 5) The palmar aponeurosis pulley is more effective than theAI pulley. Preferred Response: 1 Discussion: While cutting one of the major pulleys, A2 or A4, may result in significant changes in efficiency, a loss of both work and

excursion efficiency was noted with the loss of A4 pulley. Al and A5 are minor pulleys and show no statistical difference in work efficiency from controls if they are cut. The palmar aponeurosis may act as a pulley and should be kept intact if possible with Al pulley release for trigger fingers.

93. A 59-year-old construction worker sustained a crush injury to the volar surface of the middle phalanx and DIP joint of his index finger. The wound was repaired primarily, but the disrupted FDP tendon was not repaired. Two years later he has a 15 flexion contracture at the DIP joint with passive flexion of 25, a range of 10/95 at the PIP joint actively, and normal MP joint motion. You recommend: 1) Staged flexor tendon reconstruction. 2) One stage flexor tendon graft. 3) Flexor tenolysis. 4) FDP tendon advancement. 5) No surgical treatment. Preferred Response: 5 Discussion: This patient has an old disruption of the flexor digitorum profundus tendon and a stable but stiff DIP joint in a functional position. A prerequisite for successful outcome following flexor tendon reconstruction is full passive motion of the affected joints. Therefore, flexor tendon reconstruction is not indicated in this patient. Furthermore, he has good function of the superficialis, and late flexor tendon reconstruction through an intact superficialis may damage that tendon and jeopardize the overall function of the finger. Such reconstruction would be indicated in only selected situations. Given his stiff but well positioned DIP Joint and well functioning superficialis, no surgery is indicated.

94. The use of an above elbow thumb spica cast for six weeks

followed by a below elbow thumb spica cast until fracture healing has shown what end result in the treatment of a non-displaced scaphoid fracture. 1) Decreased time to bony union 2) Increased rate of nonunion 3) Decreased rate of malunion 4) Decreased rate of avascular necrosis 5) Increased incidence of elbow stiffness Preferred Response: 1 Discussion: Using the above treatment protocol, the time to fracture healing averaged 9.5 weeks vs. 12.7 weeks when only a below elbow thumb-spica cast was utilized. The rate of scaphoid nonunion also decreased with this protocol. These differences were more evident for proximal and mid-third fractures, distal third fractures did uniformly well regardless of the type of cast. Rates of malunion and avascular necrosis and the incidence of elbow stiffness were not influenced.

95. Which of the following inhibits contraction of Dupuytren's fibroblasts in vitro? 1) PGE, (Prostaglandin E) 2) Angiotension 3) Serotonin 4) Lysophosphatidic acid (LPA) 5) PGF2 (Prostaglandin F2) Preferred Response: 1 Discussion: Pharmacological regulation of Dupuytren's fibroblast contraction has been studied in vitro. A number of agents have been found to

either inhibit or promote contraction of these fibroblasts. Angiotension, serotonin, LPA, and PGF2 have all been shown to cause contraction. PGE2, PGEI, nifedepine, and verapamil have been shown to be inhibitors of Dupuytren's fibroblast contraction. 96. A nine-year-old boy with spastic hemiplegia has a wrist flexion deformity. Passive extension is excellent, but he is unable to actively extend his wrist. With the wrist passively held at neutral, he can fully extend his fingers actively. You recommend: 1) FCU to ECRB tendon transfer. 2) Wrist arthrodesis at neutral. 3) Flexor-pronator slide. 4) Fractional lengthening of finger flexors. 5) Tenotomy of the wrist flexors. Preferred Response: 1 Discussion: The FCU to ECRB tendon transfer helps augment wrist extensor power while reducing flexor tone at the wrist, and has been particularly effective in cases where good active finger extension is present with the wrist in a neutral position. Since the finger flexors are not excessively tight, as indicated by the ability to extend the fingers with the wrist in neutral, there is no need to release them. Fractional lengthening or flexor-pronator slide would unnecessarily eliminate useful wrist motion, and wrist flexor tenotomy could lead to unopposed wrist extension with secondary extension deformity.

97. Treatment for fingernail deformities associated with ganglions of the distal interphalangeal joint: 1) Debridement of DIP joint osteophytes. 2) Excision of the cyst wall only. 3) Intralesional injection of corticosteroid.

4) Occlusive otoform patches. 5) Removal of the nail plate cyst and osteophytes. Preferred Response: 1 Discussion: DIP joint ganglions that deform the nail and present as cystic lesions have been shown to be effectively treated by excision of the osteophyte alone. Excision of the overlying skin or cyst wall may risk injury to the germinal nail matrix and is not required. Occlusive treatment and intralesional injections will not eradicate the osteophyte deemed the most important factor in preventing recurrence.

98. Which of the following findings distinguishes compression neuropathy of the ulnar nerve at the elbow from that at the wrist? 1) Weakness of the first dorsal interosseous. 2) Decreased sensation in the ring and little fingers. 3) Decreased sensation in the dorso-ulnar hand. 4) Weakness of the profundus to the index finger. 5) Hypothenar muscle atrophy. Preferred Response: 3 Discussion: The dorsal sensory branch of the ulnar nerve innervates the dorso-ulnar hand and separates from the ulnar nerve proximal to the wrist. Thus compression of the ulnar nerve at a more proximal level, such as the elbow, can result in diminished sensation on the dorsoulnar hand, whereas more distal compression at the wrist would not. The profundus to the index finger is medianinnervated and thereby not affected by ulnar nerve compression. The other choices can all develop as the result of ulnar nerve compression at either the wrist or the elbow and therefore do not help to distinguish between these two levels.

99. When comparing intrasynovial and extrasynovial tendon autografts for flexor tendon reconstruction, the extrasynovial graft at six weeks has: 1) better gliding 2) better angular rotation at the PIP joint 3) more adhesion formation 4) better graft survival 5) Iess ultimate load Preferred Response: 3 Discussion: Canine studies have demonstrated that extrasynovial flexor tendon grafts heal by an in-growth of extrinsic adhesions. Intrasynovial grafts heal with minimal adhesions and have better graft survival which results in better gliding and PIP angular rotation at six weeks postoperatively. Extrasynovial grafts at six weeks have greater stiffness and ultimate load probably due to adhesion formation. 100. Compression of the superficial branch of the radial nerve in the distal forearm (Wartenberg syndrome) most commonly occurs between which structures? 1) Extensor carpi radialis longus and extensor carpi radialis brevis. 2) Extensor carpi radialis longus and brachioradialis. 3) Brachioradialis and flexor carpi radialis. 4) Abductor pollicis longus and extensor pollicis brevls . 5) Abductor pollicis longus and extensor pollicis longus. Preferred Response: 2 Discussion: The superficial branch of the radial nerve courses along the radial

forearm deep to the brachioradialis muscle. The nerve pierces the superficial forearm fascia near the junction of the middle and distal thirds of the forearm between brachioradialis and the extensor carpi radialis longus. It can be entrapped by either the margin of the brachioradialis or extensor carpi radialis longus or in the fascia between these two tendons.

101. A 28-year-old pipe fitter recently recovered from a distal radial fracture in his dominant hand and went back to work. He now complains of ulnar-sided wrist pain with griping, tool use, and especially with twisting activities. He is having a difficult time staying on the job. Your examination demonstrates mild swelling, but moderate tenderness over the ulnar styloid and instability of the DRUJ. Treatment should consist of: 1) Physical therapy and work hardening 2) Wrist brace 3) Open reduction and internal fixation of the ulnar styloid 4) Excision of the ulnar styloid 5) Excision of the ulnar styloid and TFCC repair Preferred Response: 5 Discussion: Because the fragment is small, it must be excised rather than repaired. Since the distal radio ulnar joint is unstable and symptomatic, simple excision is not enough. The TFCC peripheral tear must also be repaired to achieve a successful result. A brace may provide some support and relief, but does not definitively address the problem. Neither bracing nor intensive therapy will correct this problem and neither is likely to provide enough relief in the dominant hand of a skilled manual worker to allow him to return to unrestricted work within his pain tolerance. 102. What is the most common cause of compression of the ulnar nerve at the level of the wrist or through Guyon's canal?

1) Aneurysm 2) Muscle anomaly 3) Ganglion 4) Fracture 5) Thickened fascia Preferred Response: 3 Discussion: Unlike carpal tunnel syndrome where most cases are idiopathic, there is usually an identifiable cause for compression of the ulnar nerve at the wrist or through Guyon's canal. Fractures of the hook of the hamate can injure the ulnar nerve, typically the motor branch. Anomalous muscles can lead to compression, and direct trauma can lead to an acute chronic nerve injury. Thickened fascia or an abnormality of the roof of Guyon's canal have not been described. Ganglions in Guyon's canal can cause isolated sensory or motor nerve deficits or a combination of both. 103. A 52-year-old male with rheumatoid arthritis has mild wrist pain, minimal wrist deformity and a functional range of wrist motion. Radiographs show early ulnar translocation of the carpus. The surgical procedure that will arrest and correct the ulnar translocation while preserving maximum wrist function is: 1) Total wrist fusion 2) Darrach procedure 3) Radiolunate fusion 4) Sauve-Kapandji procedure 5) Lunate-capitate-hamate-triquetrum fusion Preferred Response: 3 Discussion: A total wrist fusion will eliminate ulnar translocation but will not preserve wrist function. Distal ulna procedures such as the Darrach distal ulna resection and the Sauve-Kapandji distal radio

ulnar joint reconstruction have not been shown to prevent progressive ulnar translocation. Fusion of the radius to the lunate will correct ulnar translocation while preserving some wrist motion through the mid-carpal joints. The SauveKapandji procedure may arrest ulnar translocation according to some authors, but will not correct existing ulnar translocation.

104. A 12-year-old girl with spastic hemiplegia due to cerebral palsy undergoes FCU to ECRB tendon transfer to correct a wrist flexion deformity. Postoperatively she has difficulty releasing objects. This is most likely due to: 1) Co-contraction of digital flexors and extensors. 2) Co-contraction of FCU and ECU causing wrist extension deformity. 3) Weak digital extensors. 4) Excessive tension of transferred FCU. 5) Tightness of the digital flexors. Preferred Response: 3 Discussion: The Green transfer (FCU to ECRB) is indicated in patients with wrist flexion deformity that have active digital extension with the wrist in neutral position. For those with weak digital extension that require the tenodesis effect of wrist flexion for release, this transfer is contra-indicated since release will be impaired postoperatively. 105. A 23-year-old auto mechanic sustained a crush injury of his right hand that resulted in a fracture of the second metacarpal. This was treated by ORIF using a plate and screws. The fracture heals, but the patient complains of limited index finger flexion. On examination, the PIP joint extends when the MP joint is placed in flexion. PIP flexion is normal when the MCP joint is held in neutral. This patient's problem is caused by:

1) flexor tendon adherence 2) intrinsic contracture 3) altered extensor moment arm by the plate 4) extensor tendon adherence 5) malunion of the metacarpal Preferred Response: 4 Discussion: The extrinsic extensor tendon passes across both the MP and PIP joints before inserting into the base of the middle phalanx. Thus, it can extend both the MP and PIP joints. A metacarpal fracture can cause adherence of the extensor tension at the fracture site. This tethers the extensor tendon limiting the normal tendon excursion and preventing simultaneous flexion of the MP and PIP joints. Passive MP joint flexion takes up what excursion is available and then tensions the extensor tendon causing the PIP joint to extend. Conversely, passive PIP joint flexion causes the MP to extend. These maneuvers constitute the clinical tests for extrinsic tendon adherence. 106. Exploration of a sharp transverse laceration to the volar surface of middle finger just proximal to the PIP joint demonstrates an isolated 75% laceration of the FDP tendon. Appropriate management of injury includes: 1) Wound closure; immediate mobilization program. 2) Tendon repair; immobilization for three weeks. 3) Tendon repair; immediate controlled mobilization. 4) Completion of the tendon laceration; immediate repair; immobilization for three weeks. 5) Completion of the tendon laceration, immediate repair; immediate controlled mobilization. Preferred Response: 3 Discussion:

The results of Partial Zone II flexor tendon injuries involving >60% of the cross-sectional area of the tendon are better if they are surgically repaired. Completion of the tendon laceration would disrupt what remains of the existing intrinsic blood supply of the tendon and is therefore not recommended. Numerous studies have also demonstrated the advantages of early mobilization in the treatment of Zone II flexor tendon injuries.

107. The fragment in an intra-articular fracture (Bennett type) of the thumb metacarpal is located where? 1) volar and ulnar 2) central 3) dorsal and ulnar 4) volar and radial 5) dorsal and radial Preferred Response: 1 Discussion: The Bennett "fragment" is attached to the anterior (volar) oblique ligament. This ligament is anterior and runs from the ulnar base of the metacarpal to the trapezium. It is the primary stabilizer of the trapeziometacarpal joint.

108. A 50-year-old marine biologist presents with a six-month history of pain and swelling in the proximal interphalangeal joint of his index finger (Figure 21A). Radiographs show soft-tissue swelling about the joint. A biopsy is performed (Figure 21B). Treatment should include: 1) oxacillin and an aminoglycoside 2) allopurinol 3) prednisone 4) aspirin, gold, methotrexate 5) rifampin, INH, ethambutol

Preferred Response: 5 Discussion: This patient has a granulomatous infection of the proximal interphalangeal joint which is consistent with an atypical mycobacterial infection. This man has an occupation that requires aquatic exposure and M. marinum synovitis should be suspected. Radiographs that show soft-tissue swelling but no articular destruction make a diagnosis of pyogenic infection or a noninfectious inflammatory arthropathy (like rheumatoid arthritis) unlikely. Gout histologically shows amorphous tophus with surrounding giant cells and variable inflammatory infiltrate. Atypical mycobacterial infections are granulomatous with giant cells and histocytes. Deep atypical mycobacterial infections require both surgical debridement and appropriate antibiotic therapy for nine to 12 months.

109. A six-year-old child presents for evaluation of her hands. She is somewhat obese and has had several operations on her skull to treat synostosis of the coronal, sagittal, and lamboid sutures. Examination of her hands reveals ulnar sided polydactyly, incomplete simple syndactyly of all fingers and the thumb abnormality shown in Figure 22. The most likely diagnosis is which syndrome? 1) Apert's

2) Lawrence-Moon-Biedl 3) Carpenter 4) Ellis Van Creveld 5) Down's

Preferred Response: 3 Discussion: The Carpenter syndrome was not firmly established as an entity until Temtamy's report in 1966. These patients have often been mislabeled in the literature as examples of Apert's syndrome or the LawrenceMoon-Biedl syndrome. This syndrome is thought to be inherited as an autosomal recessive condition. The characteristic features of this syndrome include obesity and duplication of the proximal phalanx of the thumb. This child also has "kissing delta" bones. Additional findings commonly include clinodactly, incomplete syndactyly, camptodactyly, and a single flexion crease. Polydactyly of the great toes, hypogenitalia, and heart defects may also be seen. Mental retardation is not an obligate feature of the syndrome and does not relate to the timing of craniofacial surgery. Synostosis of the cranial sutures is also seen in Apert's syndrome but these children never have polydactyly. Although all of the other syndromes listed may include polydactyly of the little finger or thumb except Apert's, none of these conditions is characterized by isolated duplication

of the proximal phalanges of the thumb.

110. Rerouting of the extensor pollicis longus tendon through the first dorsal compartment may be used as the sole procedure to treat mild spastic thumb-in-palm deformity. Which of the following is a contraindication to the use of EPL rerouting procedure as the sole corrective procedure. 1) Stable MCP joint 2) Adequate web space 3) Mild EPL weakness 4) Adductor pollicis contracture 5) Abductor pollicis brevis spasticity Preferred Response: 4 Discussion: Rerouting the EPL tendon through the first dorsal compartment alters its vector to coincide with that of the EPB and APL muscles. A mildly weak EPL is not a contraindication to this procedure because this transfer functions partly by a tenodesis effect. Prerequisites for a satisfactory outcome include full passive thumb abduction and no adductor or first dorsal interosseous spasticity.

111. In hands affected by rheumatoid arthritis, lateral band release (surgical separation of the lateral bands from the central slip of the extensor tendon) is indicated for the treatment of which problem? 1) Boutonniere deformity 2) Swan-neck deformity 3) Extensor tendon rupture 4) Flexor tendon rupture 5) Type II thumb deformity

Preferred Response: 2 Discussion: In swan-neck deformity without advanced joint destruction, the lateral bands may adhere to the central slip of the extensor tendon. These adherences limit volar excursion of the lateral bands during PIP flexion and thereby interfere with PIP joint flexion. Surgical separation of the lateral bands from the central slip combined with manipulation of the PIP joint into flexion and temporary pinning can improve PIP joint function in these fingers. 112. Intramuscular injection of Botulinum toxin may be indicated in which of the following: 1) 35-year-old female with myasthenia gravis and a forearm pronation contracture. 2) Prior to wrist arthrodesis in a 20-year-old male with brachial plexus palsy and a wrist flexion contracture. 3) Seven-year-old girl with cerebral palsy and a thumb adduction contracture. 4) 12-year-old boy with Werdnig-Hoffman disease and bilateral elbow flexion contractures. 5) 27-year-old quadriplegic to reduce painful spasm in the wrist extensors. Preferred Response: 3 Discussion: Botulinum toxin is a protein that reversibly blocks the release of acetylcholine in muscle and causes paralysis. It has clinical application for the reduction of muscle tone and spasm. It is used primarily in orthopaedics in the treatment of spastic conditions due to cerebral palsy and traumatic brain injury. In this case the correct potential application is in the child with a thumb adduction contracture due to cerebral palsy. None of the other situations would be appropriate for Botulinum toxin .

113. A 35-year-old banker jams his right middle finger while attempting to catch a softball bare-handed. He seeks no treatment. Two months later he complains that he cannot fully extend the DIP joint of the injured finger. Treatment should consist of: 1) repairing the terminal tendon. 2) full-time DIP joint extension splinting. 3) reconstruction of the terminal tendon. 4) observation. 5) DIP joint fusion. Preferred Response: 2 Discussion: Mallet fingers without joint subluxation even in the presence of fracture usually do extremely well with eight weeks of full-time extension splinting followed by gradual mobilization of the joint. Surgery is rarely needed and should be reserved for failure of conservative treatment. Conservative treatment is frequently successful as late as three months and on occasion even at six months following injury.

114. Which artery is the most common source of emboli of arterial origin in the upper extremity? 1) Subclavian. 2) Brachial. 3) Ulnar. 4) Radial. 5) Digital. Preferred Response: 1 Discussion: Most emboli originating from an artery start from the subclavian

artery. Signs and symptoms of a peripheral embolus are usually the sudden onset of pain followed by pallor, coldness, paresthesias, absence of distal pulses, and variable paralysis. 115. In a patient with psoriasis and arthritis mutilans affecting the interphalangeal joints of the dominant index finger, the appropriate treatment is: 1) DIP fusion and PIP joint arthroplasty 2) DIP fusion and PIP joint fusion 3) Steroid injection of DIP and PIP joints 4) Thiotepa injection of DIP and PIP joints 5) Oral methotrexate Preferred Response: 2 Discussion: Arthritis mutilans is described as severe progressive loss of bone stock with collapse and shortening of the digits that can result in the so-called "opera glass" hand. It must be treated with early and aggressive joint fusion, usually with bone graft to restore the normal length of the digit. There is no role for medical treatment in arthritis mutilans affecting the interphalangeal joints.

116. A 42-year-old healthy female is seen in clinic one week after interceding in a fight between her teenage children. She is unable to flex her ring finger on her dominant left hand. She recalled pain and swelling in the palm of her hand and finger after the fight and noted continued pain at the mid-distal palm overlying the fourth ray. There were no breaks in her skin and passive motion was intact on examination. X-rays revealed no fractures. Her diagnosis is: 1) Avulsion of the insertion of the flexor digitorum profundus to the ring finger. 2) Avulsion of the flexor digitorum superficialis of the ring finger.

3) Dislocation of the distal interphalangeal joint of the ring finger. 4) Hematoma in the flexor tendon sheath, inhibiting motion. 5) Sesamoid dislocation blocking interphalangeal joint motion. Preferred Response: 1 Discussion: Loss of distal interphalangeal joint flexion is caused by the avulsion of the flexor digitorum profundus tendon associated with painful swelling in the palm from the retracted tendon. Flexor digitorum supefficialis would not affect distal interphalangeal joint function and full passive motion is present obviating dislocation of the distal interphalangeal joint. Hematoma in the palm would not cause loss of flexion and sesamoid dislocation is unlikely and not present in view of normal X-rays. 117. Which of the following is not a cause of PIP joint contracture in Dupuytren's disease? 1) Spiral cord 2) Flexor tendon sheath 3) Retrovascular cord 4) Cleland's ligament 5) Check-rein ligaments Preferred Response: 4 Discussion: The spiral cord and retrovascular cord are direct causes of PIP joint contracture in Dupuytren's. Chronic flexion of the PIP joint results in secondary contracture of the flexor tendon sheath and check-rein ligaments of the volar plate. Cleland's ligament is not involved in Dupuytren's disease nor does it produce a secondary PIP joint contracture. 118. Distal ulnar resection in rheumatoid arthritis has been shown:

1) To have no effect on ulnar translocation of the carpus. 2) To be associated with significant carpal collapse. 3) To be associated with increased radial rotation of the carpus. 4) To diminish the development of spontaneous radiolunate fusion. 5) To increase the risk of extensor tendon rupture. Preferred Response: 1 Discussion: Rheumatoid arthritic patients who underwent excision of the distal ulna were reviewed and were noted not to have associated statistically significant collapse, ulnar translocation or radial rotation of the carpus. Sixty-one percent of the wrists spontaneously developed a radial shelf or limited radiocarpal fusion following excision of the distal ulna. Excision of the distal ulna in treatment of caput ulna syndrome would diminish the risk of extensor tendon rupture. 119. Which of the following anatomic structures is not a major contributor to the production of the spiral cord in Dupuytren's contracture. 1) Pretendinous band 2) Spiral band 3) Natatory ligament 4) Lateral digital sheet 5) Grayson's ligament Preferred Response: 3 Discussion: The spiral cord results from the progressive fibromatosis involvement of the pretendinous band, spiral band, lateral digital sheet, and Grayson's ligament. Although the natatory ligament contributes some fibers to the lateral digital sheet and may

become involved in the Dupuytren's fibromatosis producing a web space contracture, it is not involved structurally in the development of the spiral cord. 120. A 25-year-old male sustains a ring avulsion injury shown in Figure 23. On examination, the flexor digitorum sublimis is intact as is the central slip of the extensor mechanism of the finger. There are no fractures. Recommended treatment is: 1) revascularization with appropriate vein and/or artery repair. 2) amputation of the ring finger at the MCP joint level. 3) amputation of the ring finger at the PIP joint level. 4) ray amputation of the ring finger with deep transverse metacarpal ligament repair. 5) ray amputation of the ring finger with small to ring metacarpal transposition.

Preferred Response: 1 Discussion: This figure represents a Urbaniak Class II ring-avulsion injury. Such injuries may be reconstructed and satisfactory results obtained, particularly when the injury is distal to Zone 2. With a flexor digitorum sublimis and central slip of the extensor mechanism intact, satisfactory range of motion of the PIP joint can be expected. A swan-neck deformity may develop and could be reconstructed if necessary. Amputation of the ring finger at the MCP, PIP joint, or entire ray level could be considered, but this is a young, otherwise healthy individual, and therefore, revascularization is a better choice. Ray amputation and

metacarpal transposition is undesirable as it would require healing of a metacarpal which is not presently injured

Vous aimerez peut-être aussi